Está en la página 1de 158

Exámenes Selectivos para la

Olimpiada Europea de Matemática


para Mujeres (EGMO)

Comisión de Olimpiadas de la Sociedad Matemática Peruana

Edición: Jorge Tipe


Versión: febrero 2022
Prólogo

Desde el año 2018 el Perú fue invitado a participar en la Olimpiada Europea de Matemática
para Mujeres (EGMO por sus siglas en inglés). Esta olimpiada matemática de mujeres inicial-
mente estaba dirigida exclusivamente a paı́ses europeos, pero ahora es de carácter mundial.
Los paı́ses europeos tienen la categorı́a de ((oficiales)), mientras que los otros son ((invitados)),
pero las participantes de todos los paı́ses entran a una misma calificación y distribución de
medallas. En el año 2019 participaron 52 paı́ses, de los cuales 36 fueron europeos.
Según su reglamento: ((El objetivo de la EGMO es dar a más chicas una oportunidad de
desempeñarse matemáticamente en un escenario internacional y ası́ descubrir, fomentar y
retar matemáticamente a mujeres jóvenes y talentosas en todos los paı́ses europeos)).
En el Perú, la Comisión de Olimpiadas de la Sociedad Matemática Peruana está a cargo de
la selección de las alumnos, y con este fin se toman exámenes selectivos algunos meses antes
de la realización de la olimpiada.
Si encuentran un error, tienen una sugerencia para aclarar la redacción de un problema, o
si tienen cualquier otra consulta con respecto a este archivo, me pueden enviar un correo a
jorgetipe@gmail.com por lo cual estaré muy agradecido. Iré actualizando este archivo con
el paso del tiempo.

Jorge Tipe

Comisión de Olimpiadas
de la Sociedad Matemática Peruana

1
Selectivo EGMO 2021

Dı́a 1

1. Un conjunto C formado por una cantidad finita de números reales es llamado especial
si tiene al menos dos elementos y cumple la siguiente condición:
p 2b
Si a y b son elementos distintos de C entonces 5 |a| − es un elemento de C.
3
a) Determine si existe un conjunto especial que tenga dos elementos.
b) Determine si existe un conjunto especial que tenga tres o más elementos tal que
todos ellos sean positivos.

2. Encuentre todos los enteros positivos b para los cuales existe un entero positivo a con
las siguientes propiedades:

a no es un divisor de b.
aa es un divisor de bb .

3. Las rectas tangentes al circuncı́rculo del triángulo ABC que pasan por los vértices B y
C se intersectan en el punto F . Los puntos M, L y N son los pies de las perpendiculares
desde el vértice A a las rectas F B, F C y BC respectivamente.
Demuestre que
AM + AL ≥ 2AN.

4. En una mesa hay 300 manzanas y ninguna de ellas pesa más de 3 veces el peso de
cualquier otra. Demuestre que estas manzanas pueden ser divididas en grupos de 4 de
3
modo que ningún grupo pesa más de veces el peso de cualquier otro de los grupos.
2

2
Dı́a 2

5. Encuentre todos los enteros a para los cuales la ecuación


1 1 1 a
+ + =
x y z xyz

tiene infinitas soluciones enteras (x, y, z) tales que mcd(a, xyz) = 1.

6. Encuentre todas las funciones f : R → R tales que


f (x + y) ≥ xf (x) + yf (y), para todo x, y ∈ R .

7. Sean x0 , x1 , . . . , xn−1 números reales tales que 0 < |x0 | < |x1 | < . . . < |xn−1 |. Considere
la suma de los elementos de cada uno de los 2n subconjuntos de {x0 , x1 , . . . , xn−1 }.
Demuestre que los 2n números obtenidos son elementos consecutivos de una progresión
aritmética si y sólo si las razones

xi
, 0≤j <i≤n−1
xj

son iguales a las razones obtenidas con los números 20 , 21 , . . . , 2n−1 .


Nota: La suma de elementos del subconjunto vacı́o es 0.

3
Selectivo EGMO 2020

Dı́a 1

1. Para un par de conjuntos finitos A y B de números enteros no negativos, definimos


A + B como el conjunto de valores obtenidos al sumar un elemento cualquiera de A
con un elemento cualquiera de B.
Por ejemplo, si A = {2, 3} y B = {0, 1, 2, 5} entonces A + B = {2, 3, 4, 5, 7, 8}.
Determine el menor valor de k para el cual existe un par de conjuntos A y B de números
enteros no negativos con k y 2k elementos, respectivamente, tales que

A + B = {0, 1, 2, 3, . . . , 2019, 2020}.

2. Encuentre todos los pares (a, b) de números enteros tales que:


a2 + b2 es divisible por a − b − 1 ;
a2 + b 2 20
= .
2ab − 1 19

3. Sea ABC un triángulo con AB < AC y sea I su incentro. Sean M y N los puntos
medios de los lados BC y AC respectivamente. Si las rectas AI e IN son perpendicu-
lares, pruebe que la recta AI es tangente al circuncı́rculo del triángulo IM C.

4
Dı́a 2

4. Decimos que una función f : N → N es progresiva cuando cumple con las siguientes
propiedades:

f es estrictamente creciente ;
Los números a1 , a2 , a3 , . . . dados por a1 = f (1) y an+1 = f (an ) para todo n ≥ 1
están en progresión aritmética.

Determine todas las funciones progresivas f : N → N tales que f (1) = 3.

5. Sea AD el diámetro de una circunferencia ω y sea BC una cuerda de ω que es perpen-


dicular a AD. Sean M , N y P puntos en los segmentos AB, AC y BC respectivamente
tales que M P // AC y N P // AB. La recta M N corta a la recta P D en el punto Q y a
la bisectriz del ángulo ∠M P N en el punto R. Pruebe que hay una circunferencia que
pasa por los puntos B, Q, R y C.

6. Dado un tablero de 110 × 110 casillas, definimos la distancia entre dos casillas A y B
del tablero como la menor cantidad de movimientos necesarios para llevar un rey de
ajedrez desde la casilla A hasta la casilla B sin salirse del tablero.
Se tienen n casillas marcadas en el tablero de 110 × 110 de modo que la distancia entre
cualesquiera dos casillas marcadas nunca es igual a 15. ¿Cuál es el mayor valor posible
de n?

5
Selectivo EGMO 2019

Dı́a 1

1. Encuentre todos los números primos p, q y r tales que p2 + 1 = 74(q2 + r2 ).

2. Sea Γ el circuncı́rculo de un triángulo acutángulo ABC y sea H su ortocentro. La


circunferencia ω con diámetro AH corta a Γ en el punto D (D ̸= A). Sea M el punto
medio del menor arco BC ˜ de Γ. Sea N el punto medio del mayor arco BC ˜ del cir-
cuncı́rculo del triángulo BHC. Pruebe que hay una circunferencia que pasa por los
puntos D, H, M y N .

3. Para un conjunto finito A de números enteros, definimos s(A) como la cantidad de va-
lores obtenidos al sumar dos elementos cualesquiera de A, no necesariamente distintos.
De manera análoga, definimos r(A) como la cantidad de valores obtenidos al restar dos
elementos cualesquiera de A, no necesariamente distintos.
Por ejemplo, si A = {3, 1, −1} :

Los valores obtenidos al sumar dos elementos cualesquiera de A son {6, 4, 2, 0, −2}
y ası́ s(A) = 5.
Los valores obtenidos al restar dos elementos cualesquiera de A son {4, 2, 0, −2, −4}
y ası́ r(A) = 5.

Pruebe que para cada entero positivo n existe un conjunto finito A de números enteros
tal que r(A) ≥ n . s(A) .

6
Dı́a 2

4. Considere los números del 1 al 32. Se realiza un juego colocando a todos los números
en parejas y reemplazando cada pareja por el mayor divisor primo de la suma de los
números de dicha pareja. Por ejemplo, si emparejamos los 32 números ası́:

(1, 2), (3, 4), (5, 6), (7, 8), . . . , (27, 28), (29, 30), (31, 32),

obtenemos la siguiente lista de 16 números: 3, 7, 11, 5, . . . , 11, 59, 7, en donde pueden


haber repeticiones. El juego continúa de forma similar hasta que al final queda un sólo
número. Determine el mayor valor posible del número que queda al final.

5. Se define la sucesión a0 , a1 , a2 , . . . , a2018 , a2019 de números reales de la siguiente manera:


a0 = 1.
a2n
an+1 = an − para n = 0, 1, 2, . . . , 2018.
2019
1
Pruebe que a2019 < < a2018 .
2

6. Sea ABC un triángulo con AB = AC y sea M el punto medio de BC. Sea P un punto
en el plano tal que P A es paralelo a BC y P B < P C. Sean X e Y puntos en el plano
tales que B está en el segmento P X y C está en el segmento P Y . Si ∠P XM = ∠P Y M ,
pruebe que hay una circunferencia que pasa por los puntos A, P , X e Y .

7
Selectivo EGMO 2018

Dı́a 1

1. Decimos que un entero positivo n es bueno si hay tres de sus divisores (distintos dos a
dos) tales que la suma de sus cuadrados es igual a n.

a) Pruebe que todo número bueno es divisible por 3.


b) Decida si existen o no infinitos números buenos.

2. Encuentre todas las funciones f : R → R tales que


2xyf (x2 − y 2 ) = (x2 − y 2 )f (x)f (2y)

para todo par de números reales x e y.

3. Sea ABC un triángulo acutángulo cuyo circunradio R es menor que cada uno de sus
lados, sea H su ortocentro y O su circuncentro. Las bisectrices de los ángulos ∠ABH
y ∠ACH se intersectan en el punto A1 , y de manera análoga definimos también los
puntos B1 y C1 . Si E es el punto medio del segmento HO, pruebe que
3R
EA1 + EB1 + EC1 = p − ,
2
donde p es el semiperı́metro del triángulo ABC.

8
Dı́a 2

4. Sobre un tablero de 4 × 4 casillas se colocaron k bloques rectangulares de modo que:


Cada bloque cubre exactamente dos casillas.
Cada casilla del tablero quedó cubierta por al menos un bloque.
Si se retira cualquiera de los bloques, habrá al menos una casilla del tablero no
cubierta.

Encuentre el mayor valor posible de k.


Nota: Los bloques se pueden superponer.

5. Sean I el incentro del triángulo ABC e IA el excentro del triángulo ABC relativo al
lado BC. Sean M el punto medio del segmento BC y N el punto medio del arco BAC
del circuncı́rculo del triángulo ABC. T es el punto simétrico del punto N con respec-
to al punto A. Pruebe que hay una circunferencia que pasa por los puntos IA , M , I y T .

6. Encuentre todos los enteros positivos n para los cuales el número


(2n)! + 1
n! + 1
es un entero.

9
I Olimpiada Panamericana de Matemática para Mujeres
(PAGMO 2021)
Comisión de Olimpiadas de la Sociedad Matemática Peruana
y de la Pontificia Universidad Católica del Perú

Primer Examen Selectivo

Duración: 4 horas.
Está prohibido divulgar estos problemas, especialmente por internet.

1. Encuentre todos los números enteros positivos n para los cuales existen números enteros
positivos a, b y c que cumplen las tres siguientes condiciones:
a + b + c = n.
a es un divisor de b y b es un divisor de c.
a < b < c.

2. La mediatriz de la diagonal BD de un rectángulo ABCD (con AB < BC) interseca a


las rectas BC y BA en los puntos E y F , respectivamente. La recta que pasa por el
punto F y es paralela al segmento AC interseca a la recta CD en el punto G. Pruebe
que las rectas EG y AC son perpendiculares.
3. Encuentre todas las cuaternas (x, y, z, w) de números reales (no necesariamente distintos)
que resuelven el siguiente sistema de ecuaciones:
x + y = z 2 + w2 + 6zw
x + z = y 2 + w2 + 6yw
x + w = y 2 + z 2 + 6yz
y + z = x2 + w2 + 6xw
y + w = x2 + z 2 + 6xz
z + w = x2 + y 2 + 6xy

4. En cada casilla de un tablero de 2019 × 2021 casillas se escribe un número entero. Si


el número escrito en cada casilla es igual a la media aritmética de los números escritos
en dos de sus casillas vecinas, ¿cuántos números distintos escritos en el tablero pueden
haber como máximo?
Nota: Dos casillas del tablero son vecinas cuando tienen un lado común.

Nota: Recuerde enumerar las páginas de sus soluciones y procure escribir sus soluciones de
problemas distintos en páginas distintas.
San Miguel, 22 de setiembre de 2021.

1
I Olimpiada Panamericana de Matemática para Mujeres
(PAGMO 2021)
Comisión de Olimpiadas de la Sociedad Matemática Peruana
y de la Pontificia Universidad Católica del Perú

Segundo Examen Selectivo

Duración: 4 horas.
Está prohibido divulgar estos problemas, especialmente por internet.

1. Demuestre que existen infinitos enteros positivos a, b y c tales que su máximo común
divisor es 1 (es decir: mcd(a, b, c) = 1) y cumplen que:
a2 = b2 + c2 + bc.

2. Encuentre todas las funciones f : R → R tales que para cualesquiera números reales
x y y se cumple:
x2 + y 2 + 2f (xy) = f (x + y)(f (x) + f (y)).

3. En un paı́s hay 2021 ciudades. Cada pareja de ciudades se encuentran unidas por una
única carretera o no están unidas entre sı́. Se sabe que para cualquier subconjunto de
2019 ciudades el número total de carreteras entre ellas es el mismo. Si la cantidad total
de carreteras en ese paı́s es A, halle todos los posibles valores de A.

Nota: Recuerde enumerar las páginas de sus soluciones y procure escribir sus soluciones de
problemas distintos en páginas distintas.

San Miguel, 25 de setiembre de 2021.

1
Problemas de la Olimpiada
Iraní de Geometría

(Nivel Avanzado)

2014 - 2022

Juan Neyra Faustino


Fecha de edición y traducción: Diciembre 2022

JN
Segunda Olimpiada (2015)
1. Dos circunferencias ω1 y ω2 (con centro O1 y O2 respectivamente) se interse-
can en A y B. El punto X se encuentra en ω2 . Sea el punto Y un punto en ω1
tal que ∠XBY = 90◦ . La recta O1 X interseca a ω2 en X ′ por segunda vez. Si
X ′ Y interseca ω2 en K, demuestre que X se encuentra en el punto medio del
arco AK.

2. Supongamos que ABC es el triángulo equilátero con circunferencia circuns-


crita ω y circuncentro O. Sea P el punto en el arco BC (el arco en el cual A
no se encuentra). La tangente a ω en P interseca a la extensión de AB y AC
en K y L, respectivamente. Muestre que ∠KOL > 90◦ .

3. En el triángulo ABC, H es el ortocentro del triángulo. Sean l1 y l2 dos rectas


tales que pasan a través de H y son perpendiculares entre sí. La recta l1 in-
terseca a BC y a la extensión de AB en D y Z, respectivamente y la recta l2
interseca a BC y la extensión de AC en E y X, respectivamente. Dibujamos
la recta que pasa a través de D y es paralela a AC y dibujamos la recta que
pasa a través de E y es paralela a AB. Supongamos que la intersección de
estas rectas es denotada por Y . Demuestre que X, Y , Z son colineales.

4. En el triángulo ABC, dibujamos la circunferencia con centro A y radio AB.


Esta circunferencia interseca a AC en dos puntos. También dibujamos la cir-
cunferencia con centro A y radio AC y esta circunferencia interseca a AB en
dos puntos. Denote a estos cuatro puntos por A1 , A2 , A3 , A4 . Halle los pun-
tos B1 , B2 , B3 , B4 y C1 , C2 , C3 , C4 de manera similar. Supongamos que estos
12 puntos se encuentran en dos circunferencias. Demuestre que el triángulo
ABC es isósceles.

5. Los rectángulos ABA1 B2 , BCB1 C2 , CAC1 A2 se encuentran en el exterior del


triángulo ABC. Sea C ′ el punto tal que C ′ A1 ⊥ A1 C2 y C ′ B2 ⊥ B2 C1 , los puntos
A′ , B ′ se definen de manera similar. Demuestre que las rectas AA′ , BB ′ , CC ′
concurren.

1
Tercera Olimpiada (2016)
1. Las circunferencias ω y ω ′ se intersecan en A y B. La tangente a ω por A
interseca a ω ′ en C y la tangente a ω ′ por A interseca a ω en D. Suponga que
el segmento CD interseca a ω y ω ′ en E y F , respectivamente (asuma que E
está entre F y C). La perpendicular a AC que pasa por E interseca a ω ′ en el
punto P y la perpendicular a AD que pasa por F interseca a ω en el punto Q
(los puntos A, P y Q están en el mismo semiplano determinado por la recta
CD). Demuestre que los puntos A, P y Q son colineales.

2. En el triángulo acutángulo ABC, la altura desde A interseca a BC en D, y M


es el punto medio de AC. Sea X un punto tal que ∠AXB = ∠DXM = 90◦
(asuma que X y C están en semiplanos distintos determinados por la recta
BM). Demuestre que ∠XMB = 2∠MBC.

3. En un cuadrilátero ABCD, sea P la intersección de los rayos DA y CB. Sean


I1 y I2 los incentros de los triángulos P AB y P DC, respectivamente. Sean
O el circuncentro de P AB, y H el ortocentro de P DC. Demuestre que las
circunferencias circunscritas de los triángulos AI1 B y DHC son tangentes si
y sólo si las circunferencias circunscritas de los triángulos AOB y DI2 C son
tangentes.

4. En un cuadrilátero convexo ABCD, las rectas AB y CD se intersecan en E y


las rectas AD y BC se intersecan en F . Sea P el punto de intersección de las
diagonales AC y BD. Sea ω1 la circunferencia que pasa por D y es tangente a
AC en P . Sea ω2 la circunferencia que pasa por C y es tangente a BD en P . Sea
X el punto de intersección de ω1 y AD, y sea Y el punto de intersección de
ω2 y BC. Suponga que las circunferencias ω1 y ω2 se intersecan por segunda
vez en Q. Demuestre que la perpendicular desde P a la recta EF pasa por el
circuncentro del triángulo XQY .

5. ¿Existen seis puntos X1 , X2 , Y1, Y2 , Z1 , Z2 en el plano tales que todos los trián-
gulos Xi Yj Zk son semejantes para 1 ≤ i, j, k ≤ 2?

2
Cuarta Olimpiada (2017)
1. En un triángulo ABC, su circunferencia inscrita, de centro I, es tangente al
lado BC en un punto D. La recta DI corta a AC en X. La tangente por X a la
circunferencia inscrita (diferente de AC) interseca a AB en Y . Si la recta Y I
corta a BC en un punto Z, demuestre que AB = BZ.

2. Se tiene seis circunferencias que por pares no se cortan y son exteriores,


tales que cada una tiene radio mayor o igual a 1. Demuestre que el radio de
cualquier circunferencia que interseca a cada una de las seis circunferencias
dadas, es al menos 1.

3. Sea O el circuncentro del triángulo ABC. La recta CO interseca a la altura


trazada por A en el punto K. Sean P , M los puntos medios de AK, AC, res-
pectivamente. Si P O interseca a BC en Y , y la circunferencia circunscrita
del triángulo BCM corta nuevamente a AB en X, demuestre que BXOY es
cíclico.

4. Tres circunferencias ω1 , ω2 , ω3 son tangentes a una recta ℓ en los puntos A, B,


C (con B entre A y C) y ω2 es tangente externamente a las otras dos. Sean X,
Y los puntos de intersección de ω2 con la otra tangente externa común a ω1 y
ω3 . La recta perpendicular a ℓ por B, corta a ω2 nuevamente en Z. Demuestre
que la circunferencia de diámetro AC es tangente a ZX y a ZY .

5. Una esfera S es tangente a un plano. Sean A, B, C, D cuatro puntos de tal


plano de manera que no hay tres de ellos colineales. Considere el punto A′
tal que S es tangente a las caras del tetraedro A′ BCD. Los puntos B ′ , C ′ , D ′
se definen de manera similar. Demuestre que A′ , B ′ , C ′ , D ′ son coplanares y
que el plano donde se encuentran es tangente a S.

3
Quinta Olimpiada (2018)
1. Dos circunferencias ω1 , ω2 se intersecan en los puntos A, B. Sea P Q una recta
tangente común de estas dos circunferencias con P ∈ ω1 y Q ∈ ω2 . Un punto
arbitrario X se encuentra en ω1 . La recta AX interseca a ω2 por segunda vez
en Y . El punto Y ′ 6= Y se encuentra en ω2 de modo que QY = QY ′ . La recta
Y ′ B interseca a ω1 por segunda vez en X ′ . Demuestre que P X = P X ′ .

2. En el triángulo acutángulo ABC, ∠A = 45◦ . Los puntos O, H son el circuncen-


tro y el ortocentro de ABC, respectivamente. D es el pie de la altura desde B.
El punto X es el punto medio del arco AH de la circunferencia circunscrita
del triángulo ADH que contiene a D. Demuestre que DX = DO.

3. Halle todos los valores posibles de entero n > 3 tales que exista un polígono
convexo de n lados en el cual, cada diagonal es la mediatriz de al menos otra
diagonal.

4. El cuadrilátero ABCD está circunscrito alrededor de una circunferencia. Las


diagonales AC, BD no son perpendiculares entre sí. Las bisectrices de los
ángulos entre estas diagonales, intersecan a los segmentos AB, BC, CD y
DA en los puntos K, L, M y N. Dado que KLMN es cíclico, demuestre que
también lo es ABCD.

5. ABCD es un cuadrilátero cíclico. Una circunferencia que pasa por A, B es


tangente al segmento CD en el punto E. Otra circunferencia que pasa por C,
D es tangente a AB en el punto F . El punto G es el punto de intersección
de AE, DF , y el punto H es el punto de intersección de BE, CF . Demuestre
que los incentros de los triángulos AGF , BHF , CHE, DGE se encuentran en
una circunferencia.

4
Sexta Olimpiada (2019)
1. Las circunferencias ω1 y ω2 se intersecan en los puntos A y B. El punto C
se encuentra en la recta tangente desde A hacia ω1 de manera que ∠ABC =
90◦ . La recta arbitraria ℓ pasa por C e interseca a ω2 en los puntos P y Q.
Las rectas AP y AQ intersecan a ω1 por segunda vez en los puntos X y Z
respectivamente. Sea Y el pie de la altura trazada desde A hacia ℓ. Demuestre
que los puntos X, Y y Z son colineales.

2. ¿Es cierto que en cualquier polígono convexo de n lados con n > 3, existe un
vértice y una diagonal que pasa por este vértice de manera que los ángulos
de esta diagonal con ambos lados adyacentes a este vértice son agudos?

3. Las circunferencias ω1 y ω2 tienen centros O1 y O2 , respectivamente. Estas


dos circunferencias se intersecan en los puntos X y Y . AB es la recta tangen-
te común de estas dos circunferencias, de manera que A se encuentra en ω1
y B se encuentra en ω2 . Supongamos que las tangentes a ω1 y ω2 en X inter-
secan a O1 O2 en los puntos K y L, respectivamente. Suponga que la recta BL
interseca a ω2 por segunda vez en M y la recta AK interseca a ω1 por segunda
vez en N. Demuestre que las rectas AM, BN y O1 O2 concurren.

4. Dado un triángulo acutángulo escaleno ABC con circunferencia circunscrita


Γ. M es el punto medio del segmento BC y N es el punto medio de BC de
>
Ω (el que no contiene A). X y Y son puntos en Γ tales que BX k CY k AM.
Suponga que existe un punto Z en el segmento BC tal que la circunferencia
circunscrita del triángulo XY Z es tangente a BC. Suponga que ω sea la cir-
cunferencia circunscrita del triángulo ZMN. La recta AM interseca a ω por
segunda vez en P . Sea K un punto en ω tal que KN k AM, seaωb una circun-
ferencia que pasa por B, X y es tangente a BC y sea ωc una circunferencia
que pasa por C, Y y es tangente a BC. Demuestre que la circunferencia con
centro K y radio KP es tangente a las 3 circunferencias ωb , ωc y Γ.

5. Supongamos que los puntos A, B y C se encuentran en la parábola ∆ de


manera que el punto H, ortocentro del triángulo ABC, coincida con el foco
de la parábola ∆. Demuestre que al cambiar la posición de los puntos A, B y
C en ∆ de modo que el ortocentro permanezca en H, el inradio del triángulo
ABC permanece sin cambios.

5
Séptima Olimpiada (2020)
1. Sean M, N y P los puntos medios de los lados BC, AC y AB del triángulo
ABC, respectivamente. Sean E y F dos puntos en el segmento BC de modo
1 1
que ∠NEC = ∠AMB y ∠P F B = ∠AMC. Demuestre que AE = AF .
2 2
2. Sea ABC un triángulo acutángulo con su incentro I. Suponga que N es el
punto medio del arco BAC de la circunferencia circunscrita del triángulo
ABC, y P es un punto tal que ABP C es un paralelogramo. Sea Q el reflejo
de A sobre N y R la proyección de A sobre QI. Demuestre que la recta AI es
tangente a la circunferencia circunscrita del triángulo P QR.

3. Suponga tres circunferencias mutuamente fuera de cada uno con la propie-


dad de que cada recta que separa dos de ellas tiene intersección con el inte-
rior de la tercera. Demuestre
√ que la suma de las distancias por pares entre sus
centros es como máximo 2 2 veces la suma de sus radios. (Una recta separa
dos circunferencias, siempre que las circunferencias no tengan intersección
con la recta y estén en lados diferentes de la misma).

Nota. Los resultados más débiles con 2 2 reemplazados √ por alguna otra c
pueden otorgar puntos dependiendo del valor de c > 2 2.

4. El cuadrilátero circunscrito convexo ABCD con incentro I es dado de ma-


nera que su circunferencia inscrita es tangente a AD, DC, CB y BA en K,
L, M y N. Las rectas AD y BC se intersecan en E y las rectas AB y CD se
intersecan en F . Supongamos que KM interseca a AB y CD en X y Y , res-
pectivamente. Supongamos que LN interseca a AD y BC en Z y T , respec-
tivamente. Demuestre que la circunferencia circunscrita del triángulo XF Y
y la circunferencia con diámetro EI son tangentes si y solo si la circunferen-
cia circunscrita del triángulo T EZ y la circunferencia con diámetro F I son
tangentes.

5. Considere un triángulo acutángulo ABC (AC > AB) con su ortocentro H y


circunferencia circunscrita Γ. Los puntos M y P son los puntos medios de
los segmentos BC y AH, respectivamente. La recta AM interseca a Γ por se-
gunda vez en X y el punto N se encuentra en la recta BC de modo que NX
es tangente a Γ. Los puntos J y K se encuentran en la circunferencia con diá-
metro MP tal que ∠AJP = ∠HNM (B y J se encuentran en el mismo lado
de AH) y la circunferencia ω1 , que pasa a través de K, H y J, y la circunferen-
cia ω1 , que pasa a través de K, M y N, son tangentes externamente entre sí.
Demuestre que las tangentes exteriores en común de ω1 y ω2 se intersecan
en la recta NH.

6
Octava Olimpiada (2021)
1. Dado el triángulo acutángulo ABC con circunferencia circunscrita ω. Sean
D el punto medio de AC, E el pie de la altura trazada desde A hacia BC
y F el punto de intersección de AB y DE. El punto H se encuentra en el
arco BC de ω (que no contiene a A) tal que ∠BHE = ∠ABC. Demuestre que
∠BHF = 90◦ .

2. Dos circunferencias Γ1 y Γ2 se intersecan en dos puntos distintos A y B. Una


recta que pasa por A interseca de nuevo a Γ1 y Γ2 en C y D respectivamente,
de modo que A se encuentra entre C y D. La tangente en A a Γ2 interseca a
Γ1 nuevamente en E. Sea F un punto en Γ2 tal que F y A se encuentran en
lados diferentes de BD, y 2∠AF C = ∠ABC. Demuestre que la tangente en F
a Γ2 y las rectas BD y CE son concurrentes.

3. Considere un triángulo ABC con alturas AD, BE y CF , y ortocentro H. Su-


pongamos que la recta perpendicular desde H hacia EF interseca a EF , AB
y AC en P , T y L, respectivamente. El punto K está en el lado BC tal que
BD = KC. Sea ω una circunferencia que pasa por H y P , que es tangente a
AH. Demuestre que la circunferencia circunscrita del triángulo AT L y ω son
tangentes, y KH pasa por el punto de tangencia.

4. Dados 2021 puntos en el plano en posición convexa, no hay tres colinea-


les ni cuatro concíclicos. Demuestre que existen dos de ellos tales que cada
circunferencia que pase por estos dos puntos contiene al menos 673 de los
otros puntos en su interior.
(Un conjunto finito de puntos en el plano están en posición convexa si los
puntos son los vértices de un polígono convexo).

5. Dado un triángulo ABC con incentro I. La circunferencia inscrita del trián-


gulo ABC es tangente a BC en D. Sean P y Q puntos del lado BC tales que
∠P AB = ∠BCA y ∠QAC = ∠ABC, respectivamente. Sean K y L los incen-
tros de los triángulos ABP y ACQ, respectivamente. Demuestre que AD es
la recta de Euler del triángulo IKL.
(La recta de Euler de un triángulo es la recta que pasa por el circuncentro y
el ortocentro de ese triángulo).

7
Novena Olimpiada (2022)
1. Cuatro puntos A, B, C y D se encuentran en una circunferencia ω tales que
AB = BC = CD. La recta tangente a ω por el punto C interseca a la recta
tangente a ω por el punto A y a la recta AD en los puntos K y L, respecti-
vamente. La circunferencia circunscrita al triángulo KLA y ω se intersecan
nuevamente en M. Demuestre que MA = ML.

2. Sea ABC un triángulo acutángulo con AB 6= AC. Sea D un punto en la recta


BC tal que DA es tangente a la circunferencia circunscrita del triángulo ABC.
Sean E y F los circuncentros de los triángulos ABD y ACD, respectivamente,
y sea M el punto medio de EF . Demuestre que la recta tangente a la circun-
ferencia circunscrita del triángulo AMD a través de D también es tangente a
la circunferencia circunscrita del triángulo ABC.

3. En el triángulo ABC (∠A 6= 90◦ ), sean O y H el circuncentro y el pie de la altu-


ra desde A, respectivamente. Supongamos que M y N son los puntos medios
de BC y AH respectivamente. Sea D la intersección de AO y BC, y sea H ′ el
reflejo de H sobre M. Suponga que la circunferencia circunscrita al triángulo
OH ′D interseca por segunda vez a la circunferencia circunscrita a BOC en
E. Demuestre que NO y AE se intersecan en un punto de la circunferencia
circunscrita a BOC.

4. Sea ABCD un trapecio con AB k CD. Sus diagonales se intersecan en el


punto P . La recta que pasa por P paralela a AB interseca a AD y BC en Q y
R, respectivamente. Las bisectrices exteriores de los ángulos DBA y DCA se
intersecan en X. Sea S el pie de la altura trazada desde X hacia BC. Demues-
tre que si los cuadriláteros ABP Q y CDQP son circunscriptibles, entonces
P R = P S.

5. Sea ABC un triángulo acutángulo inscrito en una circunferencia ω con centro


O. Los puntos E y F se encuentran en los lados AC y AB, respectivamente,
de modo que O se encuentra en EF y BCEF es cíclico. Sean R y S las in-
tersecciones de EF con los arcos menores AB y AC de ω, respectivamente.
Supongamos que K y L son el reflejo de R sobre C y el reflejo de S sobre B,
respectivamente. Supongamos que los puntos P y Q se encuentran en las rec-
tas BS y RC, respectivamente, de modo que P K y QL son perpendiculares
a BC. Demuestre que la circunferencia de centro P y radio P K es tangente
a la circunferencia circunscrita a RCE si y solo si la circunferencia de centro
Q y radio QL es tangente a la circunferencia circunscrita a BF S.

8
Problemas de la Olimpiada
Iraní de Geometría

(Nivel Intermedio)

2014 - 2022

Juan Neyra Faustino


Fecha de edición y traducción: Diciembre 2022

JN
Primera Olimpiada (2014)
1. En un triángulo rectángulo ABC tenemos ∠A = 90◦ , ∠B = 30◦ . Denote por C a
la circunferencia que pasa a través del vértice A la cual es tangente a BC en su
punto medio. Asume que C interseca a AC y a la circunferencia circunscrita
del triángulo ABC en N y M, respectivamente. Demuestre que MN ⊥ BC.

2. En un cuadrilátero ABCD tenemos ∠B = ∠D = 60◦ . Considere la recta la


cual se dibuja desde M, el punto medio de AD, paralela a CD. Asumamos
que esta recta interseca a BC en P . Un punto X se encuentra en CD tal que
BX = CX. Demuestre que AB = BP sí y solo si ∠MXB = 60◦ .

3. Dado un triángulo acutángulo ABC. La circunferencia con diámetro BC in-


terseca a AB, AC en E, F , respectivamente. Sea M el punto medio de BC y
P el punto de intersección de AM y EF . X es un punto en el arco EF y Y
el segundo punto de intersección de XP con la circunferencia mencionada
anteriormente. Muestre que ∠XAY = ∠XY M.

4. La recta tangente a la circunferencia circunscrita del triángulo acutángulo


ABC (AC > AB) en A interseca a la continuación de BC en P . Denotemos
por O el circuncentro de ABC. X es un punto en OP tal que ∠AXP = 90◦ .
Dos puntos E, F respectivamente en AB, AC en el mismo lado de OP son
escogidos de manera que

∠EXP = ∠ACX, ∠F XO = ∠ABX.

Si K, L denota a los puntos de intersección de EF con la circunferencia cir-


cunscrita del △ABC, muestre que OP es tangente a la circunferencia circuns-
crita del △KLX.

5. Dos puntos P , Q se encuentran en el lado BC del triángulo ABC y tienen


la misma distancia al punto medio. Las perpendiculares desde P , Q hacia
BC intesectan a AC, AB en E, F , respectivamente. Sea M el punto de inter-
sección de P F y EQ. Si H1 y H2 denotan al ortocentro del △BF P y △CEQ,
muestre que AM ⊥ H1 H2 .

1
Segunda Olimpiada (2015)
1. En la figura de abajo, los puntos P , A, B se encuentran en una circunferencia.
El punto Q se encuentra en el interior de la circunferencia de modo que
∠P AQ = 90◦ y P Q = BQ. Demuestre que la resta ∠AQB menos ∠P QA es
igual al arco AB.

Q
P

2. En el triángulo acutángulo ABC, BH es la altura desde el vértice B. Los pun-


tos D y E son puntos medios de AB y AC, respectivamente. Supongamos
que F es la reflexión de H en ED. Demuestre que la recta BF pasa por el
circuncentro del △ABC.

3. En el triángulo ABC, los puntos M, N, K son los puntos medios de BC, AC,
AB respectivamente. Sean ωB y ωC dos semicircunferencias con diámetro AC
y AB fuera del triángulo respectivamente. Suponga que MK y MN intersecan
a ωC y ωB en X y Y , respectivamente. Si el punto Z es la intersección de la
tangente a ωC y ωB en X y Y respectivamente, demuestre que AZ ⊥ BC.

4. Suponga que ABC es el triángulo equilátero con circunferencia circunscrita


ω y circuncentro O. Sea P el punto en el arco BC (el arco en el cual A no
está). La recta tangente a ω en P interseca a la extensión de AB y AC en K y
L, respectivamente. Muestre que ∠KOL > 90◦ .

5. (a) ¿Existen 5 circunferencias en el plano de manera que cada circunferen-


cia pase exactamente a través del centro de 3 de otras circunferencias?
(b) ¿Existen 6 circunferencias en el plano de manera que cada circunferen-
cia pase exactamente a través del centro de 3 de otras circunferencias?

2
Tercera Olimpiada (2016)
1. En un trapecio ABCD con AB k CD, ω1 y ω2 son dos circunferencias de
diámetros AD y BC, respectivamente. Sean X y Y dos puntos arbitrarios en
ω1 y ω2 , respectivamente. Demuestre que la longitud del segmento XY no es
mayor que la mitad del perímetro de ABCD.

2. Las circunferencias C1 y C2 se intersecan en A y B. La tangente a C1 en A


interseca a C2 en P y la recta P B interseca a C1 por segunda vez en Q (suponga
que Q está afuera de C2 ). Una tangente a C2 que pasa por Q interseca a C1 y C2
en C y D, respectivamente (los puntos A y D están en diferentes semiplanos
determinados por la recta P Q). Demuestre que AD es bisectriz del ángulo
CAP .

3. Halle todos los enteros positivos N para los cuales existe un triángulo que
puede ser particionado en N cuadriláteros semejantes.

4. Sea ω la circunferencia circunscrita del triángulo rectángulo ABC (∠A = 90◦ ).


La tangente a ω por A interseca a la recta BC en el punto P . Suponga que M
es el punto medio del menor arco AB, y que P M interseca a ω por segunda
vez en Q. La tangente a ω por Q interseca a la recta AC en K. Demuestre que
∠P KC = 90◦ .

5. Las circunferencias ω y ω ′ se intersecan en A y B. La tangente a ω por A in-


terseca a ω ′ en C y la tangente a ω ′ por A interseca a ω en D. Suponga que la
bisectriz interior del ángulo CAD interseca a ω y ω ′ en E y F , respectivamente,
y la bisectriz exterior del ángulo CAD interseca a ω y ω ′ en X y Y , respecti-
vamente. Demuestre que la mediatriz de XY es tangente a la circunferencia
circunscrita del triángulo BEF .

3
Cuarta Olimpiada (2017)
1. Sea ABC un triángulo acutángulo con ∠A = 60◦ . Sean E y F los pies de las
alturas desde B y C, respectivamente. Demuestre que CE−BF = 32 (AC −AB).

2. Dos circunferencias ω1 , ω2 se intersecan en A y B. Una recta arbitraria que


pasa por B interseca a ω1 y ω2 en C y D, respectivamente. Sean E y F dos
puntos sobre ω1 y ω2 , respectivamente, tales que CE = CB y BD = DF .
Suponga que BF interseca a ω1 en P , y BE interseca a ω2 en Q. Demuestre
que A, P y Q son colineales.

3. Se tiene n puntos en el plano (n > 2), donde no hay tres de ellos colineales.
Para cada par de puntos, se dibuja la recta que pasa por ellos, y entre los
otros n − 2 puntos, se marca el punto más cercano a esta recta (considere
que en cada caso este punto es único). ¿Cuál es el máximo número posible
de puntos marcados para cada n?

4. En el triángulo isósceles ABC (AB = AC), sea ℓ la recta paralela a BC que


pasa por A. Sea D un punto arbitrario en ℓ. Sean E y F los pies de las per-
pendiculares desde A sobre BD y CD,respectivamente. Suponga que P y Q
son los pies de las perpendiculares desde E y F , respectivamente, sobre ℓ.
Demuestre que AP + AQ ≤ AB.

5. Sean X y Y dos puntos sobre el lado BC del triángulo ABC tales que 2XY =
BC (X está entre B y Y ). Sea AA′ el diámetro de la circunferencia circunscrita
del triángulo AXY . Sean P el punto de intersección de AX con la perpendi-
cular desde B a BC, y Q el punto de intersección de AY con la perpendicular
desde C a BC. Demuestre que la recta tangente desde A′ a la circunferencia
circunscrita de AXY pasa por el circuncentro del triángulo AP Q.

4
Quinta Olimpiada (2018)
1. Hay tres rectángulos en la siguiente figura. Se muestran las longitudes de
algunos segmentos. Halle la longitud del segmento XY .

4
2
6

X
5

Y
3

2. En el cuadrilátero convexo ABCD, las diagonales AC y BD se intersecan


en el punto P . Sabemos que ∠DAC = 90◦ y 2∠ADB = ∠ACB. Si tenemos
∠DBC + 2∠ADC = 180◦ , demuestre que 2AP = BP .

3. Sean ω1 , ω2 dos circunferencias con centros O1 y O2 , respectivamente. Estas


dos circunferencias se intersecan entre sí en los puntos A y B. La recta O1 B
interseca a ω2 por segunda vez en el punto C, y la recta O2 A interseca a ω1
por segunda vez en el punto D. Sea X el segundo punto de intersección de
AC y ω1 . Sea Y el segundo punto de intersección de BD y ω2 . Demuestre que
CX = DY .

4. Tenemos un poliedro cuyas caras son triángulos. Sea P un punto arbitrario


en uno de los lados de este poliedro, de manera que P no sea el punto medio
o extremo de este lado. Asume que P0 = P . En cada paso, conecte Pi al bari-
centro de una de las caras que lo contiene. Esta recta interseca al perímetro
de esta cara nuevamente en el punto Pi+1. Continuar este proceso con Pi+1
y la otra cara que contiene a Pi+1 . Demuestre que al continuar este proceso,
no podemos pasar a través de todas las caras. (El baricentro de un triángulo
es el punto de intersección de sus medianas).

5. Supongamos que ABCD es un paralelogramo tal que ∠DAC = 90◦ . Sea H


el pie de la perpendicular desde A a DC, también sea P un punto a lo largo
de la recta AC, de modo que la recta P D sea tangente a la circunferencia
circunscrita del triángulo ABD. Demuestre que ∠P BA = ∠DBH.

5
Sexta Olimpiada (2019)
1. Dos circunferencias ω1 y ω2 con centros O1 y O2 respectivamente se interse-
can entre sí en los puntos A y B, y el punto O1 se encuentra en ω2 . Sea P un
punto arbitrario que se encuentra en ω1 . Las rectas BP , AP y O1 O2 intersecan
a ω2 por segunda vez en los puntos X, Y y C, respectivamente. Demuestre
que el cuadrilátero XP Y C es un paralelogramo.

2. Encuentre todos los cuadriláteros ABCD de modo que los cuatro triángulos
DAB, CDA, BCD y ABC sean semejantes entre sí.

3. Tres circunferencias ω1 , ω2 y ω3 pasan por un punto común, digamos P . La


recta tangente a ω1 en P interseca a ω2 y ω3 por segunda vez en los puntos P1,2
y P1,3 , respectivamente. Los puntos P2,1 , P2,3 , P3,1 y P3,2 se definen de manera
similar. Demuestre que las mediatrices de los segmentos P1,2 P1,3 , P2,1 P2,3 y
P3,1 P3,2 son concurrentes.

4. Sea ABCD un paralelogramo y sea K un punto en la recta AD tal que BK =


AB. Suponga que P es un punto arbitrario en AB y que la mediatriz de P C
interseca a la circunferencia circunscrita del triángulo AP D en los puntos X,
Y . Demuestre que la circunferencia circunscrita del triángulo ABK pasa por
el ortocentro del triángulo AXY .

5. Sea ABC un triángulo con ∠A = 60◦ . Los puntos E y F son el pie de las bisec-
trices de los vértices B y C respectivamente. Los puntos P y Q se consideran
tales que los cuadriláteros BF P E y CEQF son paralelogramos. Demuestre
que ∠P AQ > 150◦. (Considere el ángulo P AQ que no contiene el lado AB del
triángulo).

6
Séptima Olimpiada (2020)
1. Dado un trapecio ABCD donde AB y CD son paralelos. Sea M el punto me-
dio del segmento AB. El punto N está ubicado en el segmento CD de manera
1 1
que ∠ADN = ∠MNC y ∠BCN = ∠MND. Demuestre que N es el punto
2 2
medio del segmento CD.

2. Sea ABC un triángulo isósceles (AB = AC) con su circuncentro O. El punto


N es el punto medio del segmento BC y el punto M es la reflexión del punto
N con respecto al lado AC. Suponga que T es un punto, de modo que ANBT
1
es un rectángulo. Demuestre que ∠OMT = ∠BAC.
2
3. En el triángulo acutángulo ABC (AC > AB), el punto H es el ortocentro y el
punto M es el punto medio del segmento BC. La mediana AM interseca a la
circunferencia circunscrita del triángulo ABC en X. La recta CH interseca a
la mediatriz de BC en E y a la circunferencia circunscrita del triángulo ABC
nuevamente en F . El punto J se encuentra en la circunferencia ω, que pasa a
través de X, E y F , de modo que BCHJ es un trapecio (CB k HJ). Demuestre
que JB y EM se intersecan en ω.

4. Dado el triángulo ABC. Una circunferencia arbitraria con centro J, que pasa
por B y C, interseca los lados AC y AB en E y F , respectivamente. Sea X un
punto tal que el triángulo F XB es semejante al triángulo EJC (con el mismo
orden) y los puntos X y C se encuentran en el mismo lado de la recta AB.
De manera similar, sea Y un punto tal que el triángulo EY C sea semejante
al triángulo F JB (con el mismo orden) y los puntos Y y B estén en el mismo
lado de la recta AC. Demuestre que la recta XY pasa por el ortocentro del
triángulo ABC.

5. Encuentre todos los números n ≥ 4 tales que exista un poliedro convexo


con exactamente n caras, cuyas caras sean triángulos rectángulos. (Tenga en
cuenta que el ángulo entre cualquier par de caras adyacentes en un poliedro
convexo es menor de 180◦ ).

7
Octava Olimpiada (2021)
1. Sea ABC un triángulo con AB = AC. Sea H el ortocentro de ABC. El punto
E es el punto medio de AC y el punto D está en el lado BC tal que 3CD = BC.
Demuestre que BE ⊥ HD.

2. Sea ABCD un paralelogramo. Los puntos E, F se encuentran en los lados


AB, CD respectivamente, tales que ∠EDC = ∠F BC y ∠ECD = ∠F AD. De-
muestre que AB ≥ 2BC.

3. Dado un cuadrilátero convexo ABCD con AB = BC y ∠ABD = ∠BCD = 90◦ .


Sea el punto E la intersección de las diagonales AC y BD. El punto F está
AF CE
en el lado AD tal que = . La circunferencia ω con diámetro DF y la
FD EA
circunferencia circunscrita del triángulo ABF se intersecan por segunda vez
en el punto K. El punto L es la segunda intersección de EF y ω. Demuestre
que la recta KL pasa por el punto medio de CE.

4. Sea ABC un triángulo acutángulo escaleno de incentro I y circunferencia


circunscrita Γ. La recta AI interseca a Γ por segunda vez en M. Sean N el
punto medio de BC y T el punto sobre Γ tal que IN ⊥ MT . Finalmente, sean
P y Q los puntos de intersección de T B y T C, respectivamente, con la recta
perpendicular a AI en I. Demuestre que P B = CQ.

5. Considere un pentágono convexo ABCDE y un punto variable X en su lado


CD. Suponga que los puntos K, L se encuentran en el segmento AX tales
que AB = BK y AE = EL y que las circunferencias circunscritas de los
triángulos CXK y DXL se intersecan por segunda vez en Y . Haciendo variar
X, demuestre que todas esas rectas XY pasan por un punto fijo, o son todas
paralelas.

8
Novena Olimpiada (2022)
1. En la siguiente figura tenemos AX = BY . Demuestre que ∠XDA = ∠CDY .

A X Y B

2. Dos circunferencias ω1 y ω2 con igual radio se intersecan en dos puntos E y


X. Los puntos arbitrarios C y D pertenecen a ω1 y ω2 , respectivamente. Las
rectas paralelas a XC y XD desde E intersecan a ω2 y ω1 en A y B, respec-
tivamente. Supongamos que CD interseca a ω1 y ω2 nuevamente en P y Q,
respectivamente. Demuestre que ABP Q es cíclico.

3. Sea O el circuncentro del triángulo ABC. Los puntos arbitrarios M y N se


encuentran en los lados AC y BC, respectivamente. Los puntos P y Q se
encuentran en el mismo semiplano que el punto C con respecto a la recta
MN y satisfacen △CMN ∼ △P AN ∼ △QMB (exactamente en ese orden).
Demuestre que OP = OQ.

4. Llamamos a dos polígonos simples P y Q compatibles si existe un entero


positivo k tal que cada uno de los polígonos P y Q se puede dividir en k
polígonos congruentes que son semejantes al otro polígono. Demuestre que
para cada dos enteros pares m, n ≥ 4, hay dos polígonos compatibles con m
y n lados.
(Un polígono simple es un polígono que no se interseca a sí mismo).

5. Sea ABCD un cuadrilátero inscrito en una circunferencia ω con centro O. Sea


P la intersección de las diagonales AC y BD. Sea Q un punto del segmento
OP . Sean E y F las proyecciones ortogonales de Q sobre las rectas AD y BC,
respectivamente. Los puntos M y N se encuentran en la circunferencia cir-
cunscrita del triángulo QEF de modo que QM k AC y QN k BD. Demuestre
que las dos rectas ME y NF se intersecan en la mediatriz del segmento CD.

9
Problemas de la Olimpiada
Iraní de Geometría

(Nivel Elemental)

2014 - 2022

Juan Neyra Faustino


Fecha de edición y traducción: Diciembre 2022

JN
Primera Olimpiada (2014)
1. En un triángulo rectángulo ABC tenemos ∠A = 90◦ , ∠C = 30◦ . Denote por C a
la circunferencia que pasa a través del vértice A la cual es tangente a BC en su
punto medio. Asume que C interseca a AC y a la circunferencia circunscrita
del triángulo ABC en N y M, respectivamente. Demuestre que MN ⊥ BC.

2. La circunferencia inscrita del △ABC es tangente a los lados BC, AC y AB en


D, E y F , respectivamente. Denote a los pies de las perpendiculares desde F ,
E a BC por K, L, respectivamente. Supongamos que la segunda intersección
de estas perpendiculares con la circunferencia inscrita son M, N, respectiva-
S
mente. Muestre que S△BM D
△CND
= DK
DL
.

3. Cada uno de Mahdi y Morteza ha dibujado un polígono de 93 lados inscrito.


Denote al primero por A1 A2 . . . A93 y al segundo por B1 B2 . . . B93 . Se sabe que
Ai Ai+1 k Bi Bi+1 para 1 ≤ i ≤ 93 (A93 = A1 , B93 = B1 ). Muestre que B Ai Ai+1
i Bi+1
es un
número constante independiente de i.

4. En un triángulo ABC tenemos que ∠C = ∠A + 90◦ . El punto D en la pro-


longación de BC es dado tal que AC = AD. Un punto E en el semiplano
determinado por la recta BC en el cual no está A es escogido de modo que
1
∠EBC = ∠A, ∠EDC = ∠A.
2

Demuestre que ∠CED = ∠ABC.

5. Dos puntos X, Y se encuentran en el arco BC de la circunferencia circuns-


crita del △ABC (este arco no contiene a A) de manera que ∠BAX = ∠CAY .
Supongamos que M denota al punto medio de la cuerda AX. Muestre que
BM + CM > AY .

1
Segunda Olimpiada (2015)
1. Tenemos cuatro triángulos de madera con lados 3, 4, 5 centímetros. ¿Cuán-
tos polígonos convexos podemos hacer con todos estos triángulos? (Solo di-
buja los polígonos sin ninguna prueba).
Un polígono convexo es un polígono cuyos ángulos son menores que 180◦ y
no tiene ningún agujero. Por ejemplo:

Este polígono no es convexo. Este polígono es convexo.

2. Sea ABC un triángulo con ∠A = 60◦ . Los puntos M, N y K están en BC, CA y


AB, respectivamente tales que BK = KM = MN = NC. Si AN = 2AK, halle
los valores de ∠B y ∠C.

3. En la figura de abajo, sabemos que AB = CD y BC = 2AD. Demuestre que


∠BAD = 30◦ .

B C
30◦

4. En el rectángulo ABCD, los puntos M, N, P y Q están en AB, BC, CD y DA,


respectivamente tales que las áreas de los triángulos AQM, BMN, CNP y
DP Q son iguales. Demuestre que el cuadrilátero MNP Q es un paralelogra-
mo.

5. ¿Existen 6 circunferencias en el plano de manera que cada circunferencia


pase exactamente a través del centro de 3 de otras circunferencias?

2
Tercera Olimpiada (2016)
1. Ali quiere moverse del punto A al punto B. No puede caminar dentro de las
áreas negras, pero puede moverse en cualquier dirección dentro de las áreas
blancas (no solo las líneas de la cuadrícula sino todo el plano). Ayuda a Ali a
encontrar el camino más corto entre A y B. Solo dibuja el camino y escribe
su longitud.

2. Sea ω la circunferencia circunscrita del triángulo ABC con AC > AB. Sea
X un punto en AC y sea Y un punto en la circunferencia ω, tal que CX =
CY = AB. (Los puntos A y Y se encuentran en diferentes lados de la recta
BC). La recta XY interseca a ω por segunda vez en el punto P . Muestre que
P B = P C.

3. Sea ABCD un cuadrilátero convexo en el que no hay dos lados paralelos.


Ubicamos los puntos P , Q, R, S tales que ABCP , BCDQ, CDAR y DABS son
paralelogramos. Demuestre que si consideramos los cuatro puntos P , Q, R,
S, exactamente uno de ellos está dentro del cuadrilátero ABCD.

4. En el triángulo rectángulo ABC (∠A = 90◦ ), la mediatriz de BC interseca a la


recta AC en K y la mediatriz de BK interseca a la recta AB en L. Si la recta
CL es la bisectriz interior del ángulo C, halle todos los posibles valores de
los ángulos B y C.

5. Sea ABCD un cuadrilátero convexo tal que ∠ADC = 135◦ y

∠ADB − ∠ABD = 2∠DAB = 4∠CBD.



Si BC = 2 · CD, demuestre que AB = BC + AD.

3
Cuarta Olimpiada (2017)
1. En un cuadrado ABCD cuyos lados tienen longitud 4, cada lado se divide en
partes iguales por medio de tres puntos. Escoge uno de los tres puntos en
cada lado, y conecta los puntos de manera consecutiva para obtener un cua-
drilátero. ¿Cuáles números pueden ser el área del cuadrilátero? Solo escribe
los números sin demostración.

A B

D C

2. Halle los ángulos del triángulo ABC.


A

/
/
/
/

/
/
B C

3. En el pentágono regular ABCDE, la perpendicular a CD que pasa por C,


interseca a AB en F . Demuestre que AE + AF = BE.
4. Sean P1 , P2 , . . . , P100 cien puntos en el plano, donde no hay tres de ellos coli-
neales. Para cada tres puntos, decimos que el triángulo formado por ellos es
positivo si cuando se leen los nombres de sus vértices desde el que tiene ín-
dice más pequeño hasta el que tiene índice más grande, se leen en el sentido
de las manecillas del reloj. ¿Es posible que el número de triángulos positivos
sea exactamente 2017?
5. En el triángulo isósceles ABC (AB = AC), sea ℓ la recta paralela a BC que
pasa por A. Sea D un punto arbitrario en ℓ. Sean E y F los pies de las per-
pendiculares desde A sobre BD y CD, respectivamente. Suponga que P y Q
son los pies de las perpendiculares desde E y F , respectivamente, sobre ℓ.
Demuestre que AP + AQ = AB.

4
Quinta Olimpiada (2018)
1. Como se muestra a continuación, hay un papel de 40 × 30 con un rectán-
gulo sombreado de 10 × 5 en su interior. Queremos recortar el rectángulo
sombreado del papel con cuatro cortes rectos. Cada corte recto es una línea
recta que divide al papel en dos piezas, y mantenemos la pieza que contiene
al rectángulo sombreado. El objetivo es minimizar la longitud total de los cor-
tes rectos. ¿Cómo lograr este objetivo y cuál es esa longitud mínima? Muestre
los cortes correctos y escribe la respuesta final. No hay necesidad de probar
la respuesta.

40

5
10
10

30
5

2. El hexágono convexo A1 A2 A3 A4 A5 A6 se encuentra en el interior del hexágono


convexo B1 B2 B3 B4 B5 B6 , de manera que A1 A2 k B1 B2 , A2 A3 k B2 B3 , . . ., A6 A1 k
B6 B1 . Demuestre que las áreas de los hexágonos simples A1 B2 A3 B4 A5 B6 y
B1 A2 B3 A4 B5 A6 son iguales. (Un hexágono simple es un hexágono que no se
interseca a sí mismo).
3. En la figura
√ dada, ABCD es un paralelogramo. Se sabe que ∠D = 60 , AD = 2

y AB = 3 + 1. El punto M es el punto medio de AD. El segmento CK es la


bisectriz de ángulo de C. Halle el ángulo CKB.

A B
K
/

?
M
/

D C

4. Existen dos circunferencias con centros O1 , O2 que se encuentran dentro de


la circunferencia ω y son tangentes a ella. La cuerda AB de ω es tangente a
estas dos circunferencias, de manera que se encuentran en lados opuestos
de esta cuerda. Demuestre que ∠O1 AO2 + ∠O1 BO2 > 90◦ .

5
5. Existen algunos segmentos en el plano que no se intersecan entre sí (incluso
en los puntos extremos). Decimos que el segmento AB rompe al segmento
CD si la extensión de AB corta a CD en algún punto entre C y D.

C
× D

(a) ¿Es posible que cada segmento cuando es extendido desde ambos ex-
tremos, rompa a exactamente otro segmento?

(b) Un segmento se llama rodeado si desde ambos lados de él existe exacta-


mente un segmento que lo rompe. (Por ejemplo, el segmento AB en la
figura). ¿Es posible que todos los segmentos sean rodeados?

×
×
B

6
Sexta Olimpiada (2019)
1. Hay una mesa en forma de rectángulo de 8 × 5 con cuatro agujeros en sus
esquinas. Después de lanzar una pelota desde los puntos A, B y C en los
caminos mostrados, ¿caerá la pelota en alguno de los agujeros después de
6 reflexiones? (La bola se refleja con el mismo ángulo después de tocar los
bordes de la mesa).

4 B 4
45 ◦
45 ◦

2
45◦
A 4
45 ◦

3 45◦
C
45◦ 1

2. Como se muestra en la figura, hay dos rectángulos ABCD y P QRD con la


misma área y con bordes paralelos correspondientes. Sean los puntos N, M
y T los puntos medios de los segmentos QR, P C y AB, respectivamente. De-
muestre que los puntos N, M y T se encuentran en la misma recta.

P Q
///

N
//

///

A D
R
M
/

T
//
/

B C

3. Existen n > 2 rectas en el plano en posición general; es decir, cualesquiera


dos de ellas se intersecan, pero no hay tres concurrentes. Todos sus puntos
de intersección son marcados, y luego se eliminan todas las rectas, pero los
puntos marcados permanecen. No se sabe qué punto marcado pertenece a
qué dos rectas. ¿Es posible saber a qué recta pertenece y restaurarlas a todas?

4. Dado el cuadrilátero ABCD de tal manera que

∠DAC = ∠CAB = 60◦ ,

7
y
AB = BD − AC.

Las rectas AB y CD se intersecan en el punto E. Demuestre que

∠ADB = 2∠BEC.

5. Para un polígono convexo (es decir, todos los ángulos son menores a 180◦ ),
llame a una diagonal bisectriz si biseca al área y al perímetro del polígono.
¿Cuál es el máximo número de diagonales que son bisectrices para un pen-
tágono convexo?

8
Séptima Olimpiada (2020)
1. Por pliegue de un papel con forma de polígono, nos referimos a dibujar un
segmento en el papel y doblar el papel a lo largo de él. Supongamos que se da
un papel con la siguiente figura. Cortamos el papel a lo largo del límite de la
región sombreada para obtener un papel con forma de polígono. Comience
con este polígono sombreado y haga un papel en forma de rectángulo con
un máximo de 5 pliegues. Describe tu solución introduciendo las líneas de
plegado y dibujando la forma después de cada doblez en tu hoja de solución.
(Tenga en cuenta que las líneas de plegado no tienen que coincidir con las
líneas de cuadrícula de la forma).

2. Dado un paralelogramo ABCD (AB 6= BC). Los puntos E y G se eligen en la


recta CD de manera que AC sea la bisectriz de ambos ángulos ∠EAD y ∠BAG.
La recta BC interseca a AE y AG en F y H, respectivamente. Demuestre que
la recta F G pasa por el punto medio de HE.
3. Según la figura, tres triángulos equiláteros con lados de longitud a, b, c tienen
un vértice común y no tienen ningún otro punto común. Las longitudes x, y
y z se definen como en la figura. Demuestre que 3(x + y + z) > 2(a + b + c).
x

b
c

y z

a
4. Sea P un punto arbitrario en el interior del triángulo ABC. Las rectas BP y
CP intersecan a AC y AB en E y F , respectivamente. Sean K y L los puntos
medios de los segmentos BF y CE, respectivamente. Las rectas que pasan
por L y K paralelas a CF y BE intersecan a BC en S y T , respectivamente;
además, denote por M y N la reflexión de S y T sobre los puntos L y K, res-
pectivamente. Demuestre que cuando P se mueve en el interior del triángulo
ABC, la recta MN pasa a través de un punto fijo.

9
5. Decimos que dos vértices de un polígono simple son visibles entre sí si son
adyacentes o si el segmento que los une está completamente dentro del po-
lígono (excepto dos extremos que se encuentran en el borde). Encuentre to-
dos los enteros positivos n tales que exista un polígono simple con n vértices
en el que cada vértice sea visible desde exactamente otros 4 vértices. (Un
polígono simple es un polígono sin agujero que no se interseca a sí mismo).

10
Octava Olimpiada (2021)
1. Al juntar las cuatro formas dibujadas en la siguiente figura, haga una forma
con al menos dos simetrías de reflexión.

2. Los puntos K, L, M, N están en los lados AB, BC, CD, DA de un cuadrado


ABCD, respectivamente, de modo que el área de KLMN es igual a la mitad
del área de ABCD. Demuestre que alguna diagonal de KLMN es paralela a
algún lado de ABCD.

3. Como se muestra en la siguiente figura, un corazón es una figura formada por


tres semicírculos con diámetros AB, BC y AC tales que B es el punto medio
del segmento AC. Dado un corazón ω. Llame a un par (P, P ′) bisectriz si P
y P ′ se encuentran en ω y bisecan su perímetro. Sean (P, P ′) y (Q, Q′ ) pares
de bisectrices. Las tangentes en los puntos P , P ′, Q y Q′ a ω construyen un
cuadrilátero convexo XY ZT . Si el cuadrilátero XY ZT está inscrito en una
circunferencia, encuentre el ángulo entre las rectas P P ′ y QQ′ .

A C
B

4. En el trapecio isósceles ABCD (AB k CD) los puntos E y F se encuentran


sobre el segmento CD de tal forma que D, E, F y C están en ese orden y
DE = CF . Sean X y Y las reflexiones de E y C con respecto a AD y AF , res-
pectivamente. Demuestre que las circunferencias circunscritas de los trián-
gulos ADF y BXY son concéntricas.

11
5. Sean A1 , A2 , . . . , A2021 2021 puntos en el plano, de modo que no hay tres co-
lineales y
∠A1 A2 A3 + ∠A2 A3 A4 + · · · + ∠A2021 A1 A2 = 360◦,

siendo el ángulo ∠Ai−1 Ai Ai+1 el que es menor que 180◦ (supongamos que
A2022 = A1 y A0 = A2021 ). Demuestre que algunos de estos ángulos suman
90◦ .

12
Novena Olimpiada (2022)
1. Halle las medidas de los ángulos del pentágono ABCDE en la siguiente figu-
ra.

B D

X Y

A E

2. Sea ABCD un trapecio isósceles con AB k CD. Los puntos E y F se en-


cuentran en los lados BC y AD, respectivamente, y los puntos M y N se
encuentran en el segmento EF de modo que DF = BE y F M = NE. Sean
K y L los pies de las perpendiculares trazadas desde M y N hacia AB y CD,
respectivamente. Demuestre que EKF L es un paralelogramo.

3. Sea ABCDE un pentágono convexo tal que AB = BC = CD y ∠BDE =


∠EAC = 30◦ . Encuentre los posibles valores de ∠BEC.

4. Sea AD una bisectriz interior del triángulo ABC, con D en BC. Las circunfe-
rencias inscritas de los triángulos ABC y ACD son tangentes exteriormente.
Demuestre que ∠ABC > 120◦ .
(Recuerde que la circunferencia inscrita de un triángulo es la circunferencia
que está dentro del triángulo y es tangente a sus tres lados).

5. a) ¿Existen cuatro triángulos equiláteros en el plano tales que cada par de


ellos tengan exactamente un vértice en común, y cada punto del plano se
encuentre en a lo mucho el borde de dos de ellos?
b) ¿Existen cuatro cuadrados en el plano tales que cada par de ellos tengan
exactamente un vértice en común y cada punto del plano se encuentre en a
lo mucho el borde de dos de ellos?
(Tener en cuenta que en ambas preguntas, no hay ninguna afirmación sobre
la intersección del interior de los polígonos).

13
Exámenes Selectivos para la
Olimpiada Matemática del Cono Sur

Comisión de Olimpiadas de la Sociedad Matemática Peruana

Edición: Jorge Tipe


Versión: julio 2022
Prólogo

En la Olimpiada Matemática de Paı́ses del Cono Sur, o simplemente Olimpiada del Cono
Sur, participan los siguientes paı́ses con una delegación de 4 alumnos: Argentina, Bolivia,
Brasil, Chile, Ecuador, Paraguay, Perú y Uruguay. Pueden participar alumnos que no hayan
cumplido 16 años al año anterior de la realización de la Olimpiada.
En el Perú, la Comisión de Olimpiadas de la Sociedad Matemática Peruana está a cargo de
la selección de los alumnos, y con este fin se toman exámenes selectivos algunos meses antes
de la realización de la olimpiada.
Si encuentran un error, tienen una sugerencia para aclarar la redacción de un problema, o
si tienen cualquier otra consulta con respecto a este archivo, me pueden enviar un correo a
jorgetipe@gmail.com por lo cual estaré muy agradecido. Iré actualizando este archivo con
el paso del tiempo. Por ejemplo, si consigo exámenes de años anteriores.

Jorge Tipe

Comisión de Olimpiadas
de la Sociedad Matemática Peruana

1
Selectivo Cono Sur 2021

Dı́a 1

1. Halle el conjunto de todos los valores posibles de la expresión ⌊m2 + 2 n⌋, donde m
y n son enteros positivos.
Nota: El sı́mbolo ⌊x⌋ denota al máximo entero menor o igual a x.

2. Para cada entero positivo k denominamos por S(k) a la suma de sus dı́gitos, por
ejemplo S(132) = 6 y S(1000) = 1. Un entero positivo n se dice que es fascinante si
k
se cumple que n = S(k) para algún entero positivo k. Por ejemplo, el número 11 es
198 198 198
fascinante puesto que 11 = S(198) (ya que S(198) = 1+9+8 = 198
18
= 11).
Pruebe que existe un entero positivo menor que 2021 y que no es fascinante.

3. Sean ABC un triángulo y D un punto en el lado BC. La recta AD interseca en el punto


E a la circunferencia circunscrita al triángulo ABC. Sean M y N los puntos medios
de los segmentos AB y CD respectivamente. Los segmentos M N y AD se intersecan
en el punto F . Sea G (distinto de F ) el punto de intersección de las circunferencias cir-
cunscritas a los triángulos DN F y ECF . Pruebe que los puntos B, F y G son colineales.

4. Sea n ≥ 5 un número entero. Considere 2n − 1 subconjuntos A1 , A2 , A3 , ..., A2n−1 del


conjunto {1, 2, 3, ..., n}, estos subconjuntos tienen la propiedad de que cada uno de ellos
tiene 2 elementos (es decir, para 1 ≤ i ≤ 2n − 1 se cumple que Ai tiene 2 elementos).
Pruebe que siempre es posible seleccionar n de estos subconjuntos de manera que la
unión de estos n subconjuntos tenga a lo más 23 n + 1 elementos en total.

2
Dı́a 2

5. Sea n ≥ 2 un número entero. Son dados n + 1 puntos rojos en el plano.


Pruebe que existen 2n circunferencias C1 , C2 , ..., Cn , D1 , D2 , ..., Dn tales que:

C1 , C2 , ..., Cn son concéntricas.


D1 , D2 , ..., Dn son concéntricas.
Para k = 1, 2, 3, ..., n las circunferencias Ck y Dk son disjuntas.
Para k = 1, 2, 3, ..., n se cumple que Ck contiene exactamente k puntos rojos en
su interior y Dk contiene exactamente n + 1 − k puntos rojos en su interior.

6. Pruebe que no existen enteros positivos a1 , a2 , ..., a2021 (no necesariamente distintos)
tales que para k = 1, 2, 3, ..., 2021 el número de elementos del conjunto

Ak = { j ∈ N : 1 ≤ j ≤ 2021 y aj |k }

sea exactamente ak .

7. Sea n un entero positivo. Sea A un subconjunto de {0, 1, 2, 3, ..., 5n } tal que A tiene
4n + 2 elementos. Pruebe que existen a, b y c elementos de A tales que a < b < c y
c + 2a > 3b.

3
Selectivo Cono Sur 2020

Dı́a 1

1. En un salón de clase hay m estudiantes. Durante el mes de julio cada uno de ellos visitó
la biblioteca al menos una vez pero ninguno de ellos visitó la biblioteca dos veces en
el mismo dı́a. Resultó que durante el mes de Julio cada estudiante visitó la biblioteca
un número distinto de veces, además para cualesquiera dos estudiantes A y B hubo un
dı́a en el cual A visitó la biblioteca y B no lo hizo y también hubo un dı́a cuando B
visitó la biblioteca y A no lo hizo. Determine el mayor valor posible de m.
Nota: El mes de julio tiene 31 dı́as.

2. Sea Z el conjunto de los números enteros. Encuentre todas las funciones f : Z → Z


que satisfacen las condiciones:

(i) f (f (x)) = xf (x) − x2 + 2 para todo x ∈ Z.


(ii) f toma todos los valores enteros.

3. Sea ABC un triángulo acutángulo con AB > AC. Sea D el pie de la altura desde A
hacia BC, sea K la intersección de AD con la bisectriz interna del ángulo B, sea M
el pie de la perpendicular de B hacia CK (podrı́a encontrarse en la prolongación del
segmento CK) y N la intersección de BM y AK (podrı́a encontrarse en la prolonga-
ción de los segmentos). Sea T la intersección de AC con la recta que pasa por N y es
paralela a DM . Pruebe que BM es la bisectriz interna del ángulo ∠T BC.

4. Halle todos los enteros positivos impares n para los cuales


2ϕ(n) − 1
n
es un cuadrado perfecto.

Nota: Dado un entero positivo n, ϕ(n) denota a la cantidad de elementos del conjunto:

{a ∈ Z : 1 ≤ a ≤ n y mcd(a, n) = 1}.

(∗) En el año 2020 el primer examen selectivo para la Olimpiada del Cono Sur coincidió con el de la
Olimpiada Iberoamericana.

4
Dı́a 2

5. Encuentre el menor entero positivo n de modo que para cualesquiera n números reales
distintos b1 , b2 , ..., bn en el intervalo [1, 1000] siempre existan bi y bj tales que:
p
0 < bi − bj < 1 + 3 3 bi bj

6. Sea a1 , a2 , a3 , . . . una secuencia de enteros positivos satisfaciendo las siguientes condi-


ciones:
a1 = 1, an+1 = an + a⌊√n⌋ para todo n ≥ 1
Pruebe que para cada entero positivo k existe un término ai que es divisible por k.

Nota: El sı́mbolo ⌊x⌋ denota al mayor número entero que es menor o igual a x.

7. Sea n ≥ 2. Ana y Beto juegan el siguiente juego: Ana escoge 2n números reales no
negativos x1 , x2 , . . . , x2n (no necesariamente distintos) cuya suma total es 1, y los mues-
tra a Beto. Luego Beto ordena estos números en una circunferencia de la manera que
crea conveniente, calcula el producto de cada par de números adyacentes y escribe el
máximo valor de estos productos. Ana quiere maximizar el número escrito por Beto,
mientras que Beto quiere minimizarlo.
¿Que número se escribirá si ambos juegan de manera óptima?

5
Selectivo Cono Sur 2019

Dı́a 1

1. Encuentre todos los números enteros positivos a y b tales que


ab + b a
aa − b b
es un número entero.

2. Sea AB el diámetro de una circunferencia Γ con centro O. Sea CD una cuerda de Γ


que es perpendicular a AB y sea E el punto medio de CO. La recta AE corta a Γ en
un punto F (F ̸= A). El segmento BC corta a los segmentos AF y DF en los puntos
M y N respectivamente. El circuncı́rculo del triángulo DM N corta a Γ en un punto
K (K ̸= D). Pruebe que KM = M B.

3. Sea A el número de formas en que se puede particionar el conjunto {1, 2, . . . , n} en


subconjuntos no vacı́os. Sea B el número de formas en que se puede particionar el
conjunto {1, 2, . . . , n, n + 1} en subconjuntos no vacı́os tales que números consecutivos
pertenezcan a subconjuntos distintos. Particiones que solamente difieren por el orden
de los subconjuntos se consideran iguales. Pruebe que A = B.

6
Dı́a 2

4. En las casillas de un tablero 3 × 3 se escriben los números del 1 al 9. Se define una


operación del siguiente modo:
Tome una fila o columna cualquiera y reemplace los números a, b, c en ella por los
números no negativos a − x, b − x, c + x o a + x, b − x, c − x (en el mismo orden),
donde x es un real positivo que puede variar en cada operación.

a) ¿Es posible, luego de una serie de operaciones en los tableros

1 2 3 2 8 5
4 5 6 9 3 4
7 8 9 6 7 1

obtener todos los números iguales en cada uno de ellos?


b) Luego de una serie de operaciones podemos obtener desde un tablero inicial todos
los números iguales a un cierto número m. ¿Cuál es el máximo valor que puede
tomar m?

5. En una pizarra está escrito el número racional q. Marı́a puede borrarlo y sustituirlo
q−1  1 1
por q + 1, por q − 1 o por si q ̸= . Marı́a debe obtener el número tras
2q − 1 2 2020
un número finito de operaciones.

a) Pruebe que si inicialmente q = 0, Marı́a nunca podrá conseguir su objetivo.


b) Encuentre todos los valores iniciales de q para los que Marı́a puede conseguir su
objetivo.

6. Dos polinomios del mismo grado A(x) = an xn + · · · + a1 x + a0 y B(x) = bn xn +


· · · + b1 x + b0 (an , bn ̸= 0) se llaman amigos si los coeficientes b0 , b1, , . . . , bn son una
permutación de los coeficientes a0 , a1, , . . . , an .
Sean P (x) y Q(x) dos polinomios amigos con coeficientes enteros. Si P (16) = 32020 ,
determine el menor valor posible de |Q(32020 )|.

7
Selectivo Cono Sur 2018

Dı́a 1

1. Considere 2016 puntos distintos sobre una circunferencia. Está permitido ir de un punto
a otro sobre la circunferencia saltando 2 o 3 puntos adelante en sentido horario. ¿Cuál
es el mı́nimo número de saltos requeridos para visitar a todos los puntos y regresar al
punto de inicio?

2. Sea x un número real positivo tal que los números x−1 , x y x2018 tienen la misma parte
fraccionaria:
{x−1 } = {x} = {x2018 }.
Pruebe que x = 1.
Nota: Si x es un número real, la parte fraccionaria de x es {x} = x − ⌊x⌋, donde ⌊x⌋
denota al mayor entero que es menor o igual que x.

3. Sea I el incentro de un triángulo ABC con AB ̸= AC y sea M el punto medio del arco
BAC del circuncı́rculo de dicho triángulo. La recta perpendicular a AI que pasa por
I intersecta a la recta BC en el punto D. La recta M I intersecta al circuncı́rculo del
triángulo BIC en el punto N . Pruebe que la recta DN es tangente al circuncı́rculo del
triángulo BIC.

4. Considere los números


1 1 1 1
S1 = + + + ··· + ,
1·2 1·3 1·4 1 · 2018
1 1 1 1
S2 = + + + ··· + ,
2·3 2·4 2·5 2 · 2018
1 1 1 1
S3 = + + + ··· + ,
3·4 3·5 3·6 3 · 2018
.. ..
. .
1
S2017 = .
2017 · 2018

Pruebe que el número S1 + S2 + S3 + . . . + S2017 no es entero.

8
Dı́a 2

5. Halle todos los enteros positivos d que pueden escribirse en la forma


d = mcd |x2 − y| , |y 2 − z| , |z 2 − x| ,


donde x, y, z son enteros positivos coprimos dos a dos tales que x2 ̸= y, y 2 ̸= z, z 2 ̸= x.

6. Sea n un entero positivo. En un tablero de n × n casillas se han pegado dos lados


opuestos formando ası́ un cilindro. Decida si es posible o no colocar n reinas en el
tablero de modo que no hayan dos de ellas que se amenacen cuando:

a) n = 14;
b) n = 15.

7. Sea ABCD un cuadrado fijo y K un punto variable en el segmento AD. Se construye


el cuadrado KLM N de modo que B está en el segmento LM y C está en el segmento
M N . Sea T el punto de intersección de las rectas LA y N D. Halle el lugar geométrico
de T conforme K varı́a en el segmento AD.

9
Dı́a 3

8. Para cada pareja de enteros positivos m y n, definimos fm (n) de la siguiente manera:


fm (n) = mcd(n, d1 ) + mcd(n, d2 ) + · · · + mcd(n, dk ) ,

donde 1 = d1 < d2 < · · · < dk = m son todos los divisores positivos de m. Por ejemplo,
f4 (6) = mcd(6, 1) + mcd(6, 2) + mcd(6, 4) = 5.

a) Encuentre todos los enteros positivos n tales que f2017 (n) = fn (2017).
b) Encuentre todos los enteros positivos n tales que f6 (n) = fn (6).

9. Encuentre el mayor valor posible del entero positivo N si se sabe que hay enteros
positivos a1 , a2 , . . . , aN tales que
p
an = (an−1 )2 + 2018 an−2 para n = 3, 4, . . . , N .

10. Sea n un entero positivo. Alex juega en una fila de 9 casillas del siguiente modo. Inicial-
mente, todas las casillas están vacı́as. En cada turno, Alex debe realizar exactamente
una de las siguientes jugadas:

i) Elige un número de la forma 2j , con j entero no negativo, y lo coloca en una


casilla vacı́a;
ii) Elige dos (no necesariamente consecutivas) casillas con el mismo número escrito
en ellas, digamos 2j , reemplaza el número en una de las casillas por 2j+1 y borra
el número en la otra casilla.

Al final del juego, una casilla contiene el número 2n , mientras que las otras casillas
están vacı́as. Determine, en función de n, el máximo número de turnos realizados por
Alex.

10
Selectivo Cono Sur 2017

Dı́a 1

1. Decimos que una diagonal de un pentágono convexo es buena si divide al pentágono en


un triángulo y en un cuadrilátero circunscriptible. ¿Como máximo cuantas diagonales
buenas puede tener un pentágono convexo?
Aclaración: Un cuadrilátero es circunscriptible si existe una circunferencia que es tan-
gente a sus cuatro lados.

2. Determine si existe un entero positivo n tal que 10n − 1 es un divisor de 11n − 1.

3. Un L-trominó es una figura formada por tres cuadraditos que se obtiene al eliminar un
cuadradito de un tablero de 2 × 2.
Se tiene un tablero de 7 × 7 formado por 112 segmentos unitarios. Una configuración
de algunos L-trominós es óptima si los L-trominós no se superponen, cada uno cubre
exactamente tres cuadraditos del tablero y además ningún segmento unitario del ta-
blero pertenece a dos L-trominó. A continuación se muestra una configuración óptima
de 5 L-trominós:

Determine el mayor valor posible de n para el cual existe una configuración óptima de
n L-trominós en el tablero de 7 × 7.

4. Sea n un entero positivo fijo. Hallar la mayor constante real Cn que tiene la siguiente
propiedad: Cualesquiera 2n números reales, no necesariamente distintos, que pertene-
cen al intervalo [100, 101] se pueden particionar en dos grupos con sumas S1 y S2 tales
que
S2
1≥ ≥ Cn .
S1

11
Dı́a 2

5. Sea ABC un triángulo acutángulo de circuncentro O. Se traza la altura BQ, con Q en


el lado AC. La paralela a la recta OC que pasa por Q intersecta a la recta BO en X.
Pruebe que X y los puntos medios de los lados AB y AC son colineales.

6. Sean n y ℓ enteros positivos con ℓ > 7. Hay n fichas en la casilla del extremo izquierdo
de una fila (horizontal) de ℓ casillas. Una jugada consiste en mover cualquier ficha 1,
2, 3, 4, 5 o 6 posiciones hacia la derecha. Andrés y Beto realizan jugadas por turnos y
Andrés empieza. El ganador es el que ubica una ficha en la casilla del extremo derecho.
Determine quién tiene estrategia ganadora en función de n y ℓ.

7. Para cada entero positivo n, definimos Pn = (n + 1)(n + 2)(n + 3) · · · (n + 2016) y


Qn = mcm(n + 1, n + 2, n + 3, . . . , n + 2016), es decir, Qn es el mı́nimo común múltiplo
de los números n + 1, n + 2, n + 3, . . . , n + 2016. Determine si existe o no una constante
Pn
C tal que < C, para todo entero positivo n.
Qn

12
Dı́a 3

8. Determine el menor entero positivo n para el cual la siguiente proposición es verdadera:


Si los números enteros positivos a y b son tales que a + b es múltiplo de 100 y ab es
múltiplo de n, entonces cada uno de los números a y b es múltiplo de 100.

9. Sea BXC un triángulo y A1 , A2 , A3 puntos del mismo plano tales que X es el ortocen-
tro de A1 BC, X es el incentro de A2 BC y X es el baricentro de A3 BC. Si A1 A3 es
paralelo a BC, pruebe que A2 es el punto medio de A1 A3 .

10. Miguel tiene una lista de varios subconjuntos de 10 elementos de {1, 2, . . . , 100}. Él le
dice a Cecilia: ((Si eliges cualquier subconjunto de 10 elementos de {1, 2, . . . , 100}, será
disjunto con al menos un subconjunto de mi lista.)) ¿Cuál es la mı́nima cantidad de
subconjuntos que puede tener la lista de Miguel, si lo que le dice a Cecilia es cierto?

13
Selectivo Cono Sur 2016

Dı́a 1

1. ¿Cuántos múltiplos de 11 de cuatro dı́gitos, de la forma abcd, cumplen que a ̸= b, b ̸= c


y c ̸= a?

2. Sea ω una circunferencia. Para cada n, sea An el área de un polı́gono regular de n lados
circunscrito a ω y Bn el área de un polı́gono regular de n lados inscrito en ω. Pruebe
que 3A2015 + B2015 > 4A4030 .

3. Diez alumnos están sentados alrededor de una mesa circular. La profesora tiene una
lista de quince problemas y a cada alumno le entrega seis problemas, de tal modo
que cada problema es entregado exactamente cuatro veces y cualesquiera dos alumnos
tienen a lo más tres problemas en común. Pruebe que, sin importar cómo la profesora
distribuya los problemas, siempre habrá dos alumnos sentados uno al lado del otro que
tengan al menos un problema en común.

14
Dı́a 2

4. Sea n un entero positivo. Andrés tiene n + 1 tarjetas y cada una de ellas tiene escrito
un entero positivo, de tal forma que la suma de los n + 1 números es 3n. Pruebe que
Andrés puede ubicar una o más tarjetas en una caja roja y una o más tarjetas en una
caja azul, de tal forma que la suma de los números de las tarjetas de la caja roja sea
igual al doble de la suma de los números de las tarjetas de la caja azul.
Aclaración: Algunas tarjetas de Andrés pueden quedar fuera de las cajas.

5. Encuentre todos los enteros positivos n para los cuales 2n + 2021n es un cuadrado
perfecto.

6. Dos circunferencias ω1 y ω2 , que tienen centros O1 y O2 , respectivamente, se intersec-


tan en A y B. Una recta ℓ que pasa por B corta a ω1 nuevamente en C y corta a
ω2 nuevamente en D, de tal forma que los puntos C, B, D aparecen en ese orden. Las
tangentes a ω1 y ω2 en C y D, respectivamente, se intersectan en E. La recta AE in-
tersecta nuevamente a la circunferencia circunscrita del triángulo AO1 O2 en F . Pruebe
que la longitud del segmento EF es constante, es decir, no depende de la elección de
ℓ.

15
Selectivo Cono Sur 2015

Dı́a 1

1. A escribe, a su elección, 8 unos y 8 dos en un tablero de 4 × 4. Luego B cubre el tablero


con 8 dominós y para cada dominó halla el menor de los dos números que cubre ese
dominó. Finalmente, A suma estos 8 números y el resultado es su puntaje. ¿Cuál es el
mayor puntaje que A se puede asegurar, sin importar cómo juegue B?
Aclaración: Un dominó es un rectángulo de 1 × 2 o de 2 × 1 que cubre exactamente
dos cuadraditos del tablero.

2. Sean a, b, c y d elementos del conjunto {1, 2, 3, . . . , 2014, 2015} tales que a < b < c < d,
a + b es un divisor de c + d y a + c es un divisor de b + d. Determine el mayor valor
que puede tomar a.

3. Sea ABCD un paralelogramo. Se elige un punto X del lado AB y un punto Y del lado
CD. Los segmentos AY y DX se cortan en P ; y los segmentos BY y CX se cortan en
Q. Pruebe que la recta P Q pasa siempre por un punto fijo, sin importar la elección de
los puntos X y Y .

4. En una pequeña ciudad hay n rutas de buses, con n > 1, y cada ruta tiene exactamente
4 paraderos. Si cualesquiera dos rutas tienen exactamente un paradero en común, y
cada pareja de paraderos pertenece a exactamente una ruta, halle todos los posibles
valores de n.

16
Dı́a 2

5. Halle el menor término de la sucesión a1 , a2 , a3 , . . . definida por a1 = 201420152016 y


( an
si an es par,
an+1= 2
an + 7 si an es impar.

6. Sea n un entero positivo. En un tablero de 2n × 2n, 2n2 casillas se pintaron de blanco


y las otras 2n2 , de negro. Una operación consiste en escoger un subtablero de 2 × 2
y reflejar sus 4 casillas con respecto al eje de simetrı́a vertical u horizontal de dicho
subtablero. ¿Para qué valores de n es posible siempre conseguir la coloración similar
al ajedrez, a partir de cualquier coloración inicial?

7. En el plano se ubicaron 6 puntos tales que la distancia entre dos cualesquiera de ellos
es mayor o igual que 1. Pruebe que es posible escoger dos de esos puntos tales que su
distancia sea mayor o igual que 2 cos 18◦ .
Observación: Le podrı́a ser de ayuda saber que cos 18 = 0,95105 . . . y cos 24 = 0,91354 . . ..

17
Dı́a 3

8. Sea ABCD un cuadrilátero cı́clico tal que los rayos AB y DC se intersectan en K.


Sean M y N los puntos medios de los segmentos AC y KC, respectivamente. Halle
todos los posibles valores de ∠ADC si los puntos M , B, N y D pertenecen a una
misma circunferencia.

9. Sean m y n enteros positivos. Un niño recorre el plano cartesiano dando algunos pasos.
El niño comienza su recorrido en el punto (0, n) y termina en el punto (m, 0) de tal
forma que:

Cada paso tiene longitud 1 y es paralelo al eje X o al eje Y .


Para cada punto (x, y) de su recorrido se cumple que x ≥ 0 y y ≥ 0.

Para cada paso del niño se calcula la distancia que hay entre el niño y el eje al cual
es paralelo dicho paso. Si el paso hace que el niño esté más lejos del punto (0, 0) que
antes, consideramos esa distancia como positiva, caso contrario, consideramos esa dis-
tancia como negativa. Pruebe que al finalizar el recorrido del niño, la suma de todas
las distancias es 0.

10. Sea n un entero positivo. Se tiene una colección de tarjetas que cumple las siguientes
propiedades:

Cada tarjeta tiene escrito un número de la forma m!, donde m es un entero


positivo.
Para todo entero positivo t ≤ n!, es posible escoger una o más tarjetas de la
colección de tal forma que la suma de los números de esas tarjetas sea t.

Determine, en función de n, el menor número de tarjetas que puede tener dicha colec-
ción.

18
Selectivo Cono Sur 2014

Dı́a 1
1 1
1. Un par ordenado (a, b) de enteros positivos es llamado decianimal cuando + es
a b
m
igual a una fracción decimal , con mcd(m, 10) = 1. Halle todos los pares decianima-
10
les.

2. Sea ABCD un cuadrilátero cı́clico. Suponga que los rayos BC y AD se intersectan en


el punto P y Q es un punto del plano tal que P es punto medio del segmento BQ.
Se construyen los paralelogramos CAQR y DBCS. Pruebe que los puntos C, Q, R, S
pertenecen a una misma circunferencia.

3. Dado un arreglo de n números reales, podemos realizar varias veces la siguiente ope-
ración: Elegir un número primo p ≤ n, y p de los n números, para luego reemplazar
cada uno de ellos por el promedio aritmético de los p números. El objetivo es que al
final todos los n números sean iguales.

a) Si n = 2014, probar que bastan 1151 operaciones para conseguir el objetivo, sea
cual sea el arreglo inicial.
b) Si n ≤ 2014, probar que bastan 7744 operaciones para conseguir el objetivo, sea
cual sea el arreglo inicial.

4. Sean P1 , P2 , . . . , Pn n puntos diferentes alrededor de una circunferencia. Se une cada


par de puntos por medio de un segmento que es coloreado de rojo o azul. Considere
una coloración para la cual Pi Pj es rojo si y sólo si Pi+1 Pj+1 es azul, para cualesquiera
ı́ndices distintos i, j en el conjunto {1, 2, . . . , n} (asumimos que Pn+1 = P1 ).

a) ¿Para qué valores de n es posible tal coloración?


b) Si un paso consiste en moverse a lo largo de un segmento rojo, desde un extremo
al otro, demuestre que es posible ir desde cualquier punto Pi hasta cualquier otro
punto Pj en a lo más tres pasos.

19
Dı́a 2

5. Determine todos los enteros positivos n ≥ 4 que satisfacen la siguiente propiedad: Si los
números reales no nulos a1 , a2 , . . . , an cumplen que para cualesquiera tres subı́ndices
1 ≤ i < j < k ≤ n existe un subı́ndice ℓ distinto de i, j, k tal que ai · aj · ak = a3ℓ ,
entonces a1 = a2 = · · · = an .

6. Marı́a puede elegir un número positivo ℓ < 1 y varios cuadrados de lado ℓ para cubrir
un cuadrado de lado 1. ¿Cuál es el menor número de cuadrados que necesita usar Marı́a?

7. Una sucesión estrictamente creciente e infinita de enteros positivos es n-elegante si se


cumplen las siguientes dos condiciones:

cualesquiera dos términos de la sucesión son coprimos,


la suma de los digitos de cada término es n.

Demostrar que hay infinitos enteros positivos n para los cuales es posible encontrar
una sucesión n-elegante.

20
Dı́a 3

8. Sea Γ un cı́rculo y A un punto exterior a Γ. Las rectas tangentes a Γ que pasan por A
tocan a Γ en B y C. Sea M el punto medio de AB. El segmento de recta M C corta
a Γ nuevamente en D y la recta AD corta a Γ nuevamente en E. Siendo AB = a y
BC = b, hallar CE en términos de a y b.

9. Hallar el mayor entero positivo n para el cual existe una sucesión a0 , a1 , a2 , . . . , an de ci-
fras no nulas (es decir, ai ∈ {1, 2, . . . , 9}) tal que el número de k cifras ak−1 ak−2 . . . a1 a0
divide al número de k + 1 cifras ak ak−1 . . . a1 a0 para todo k, 1 ≤ k ≤ n.

10. En un torneo de ajedrez cada dos jugadores han jugado exactamente un partido. La
victoria vale 1 punto, la derrota 0, y en caso de empate cada jugador obtiene 1/2
punto. Un partido es llamado anómalo si el ganador de ese partido, al finalizar el
torneo, obtuvo menos puntaje que el perdedor de ese partido.

a) ¿Es posible que más del 75 % del total de partidos sean anómalos?
b) ¿Es posible que más del 70 % del total de partidos sean anómalos?

21
Selectivo Cono Sur 2013

Dı́a 1

1. Dos piedras, una blanca y una negra, están ubicadas en dos casillas de un tablero de
ajedrez (de 8 × 8). En cada movida una de las piedras se mueve a una casilla vecina, de
tal forma que en ningún momento las dos piedras están en la misma casilla. Determine
si es posible o no que, después de una secuencia de movidas, cada forma de ubicar a
las piedras sobre el tablero haya aparecido exactamente una vez.
Aclaración: Dos casillas son vecinas si comparten un lado.

2. Dado un triángulo ABC, sean M , N y P puntos de los lados AB, BC y CA, respecti-
vamente, tales que M BN P es un paralelogramo. La recta M N corta a la circunferencia
circunscrita del triángulo ABC en los puntos R y S. Pruebe que la circunferencia cir-
cunscrita del triángulo RP S es tangente a AC.

3. Pruebe que, para cada entero impar n > 1, existen tres enteros positivos a, b, c, copri-
mos entre sı́ (dos a dos), tales que

a2 + 2b2 + 4c2 = 3n .

(John Cuya)

22
Dı́a 2

4. Sean n, a, b enteros positivos, con a > b, tales que n2 + 1 = ab.


a) Pruebe que

a−b≥ 4n − 3. (∗)

b) Halle todos los enteros positivos n para los cuales puede ocurrir la igualdad en (∗).

5. Sea I el incentro del triángulo ABC y sean A1 , B1 y C1 puntos que pertenecen a


los segmentos AI, BI y CI, respectivamente. Las mediatrices de los segmentos AA1 ,
BB1 y CC1 determinan un triángulo T . Si I es el ortocentro del triángulo A1 B1 C1 ,
demuestre que los circuncentros de los triángulos T y ABC coinciden.

6. Un club de caminata con 4n miembros organiza una serie de caminatas a lo largo de


cierto número de fines de semana, de acuerdo a las siguientes reglas:

a) Cada fin de semana hay dos caminatas: una en el dı́a sábado y la otra en el dı́a
domingo.
b) Exactamente 2n miembros del club participan en cada caminata.
c) En cada fin de semana ningún miembro participa en las dos caminatas.
d ) Después de que todas las caminatas hayan concluido, cualesquiera dos miembros
del club han participado juntos en r caminatas (r es un número fijo).

Pruebe que después de que todas las caminatas hayan concluido, cualesquiera tres
miembros del club han participado juntos en t caminatas, donde t es un número fijo
que es múltiplo de n − 1.

23
Dı́a 3

7. Determine el mayor número real c que tiene la siguiente propiedad: En cualquier


heptágono convexo la suma de las longitudes de todas sus diagonales es mayor que cP ,
donde P es el perı́metro del heptágono.

8. Sea A un conjunto finito, formado por enteros positivos. Decimos que A es bueno si
cumple las siguientes dos propiedades:

Para cualesquiera tres elementos distintos a, b, c de A, se cumple que su máximo


común divisor es 1.
Para cualesquiera dos elementos distintos b y c de A, existe un elemento a de A
tal que a ̸= b, a ̸= c y a | bc.

Determine todos los conjuntos buenos.

9. La secuencia n1 , n2 , . . . , n2013 de enteros positivos cumple que ni ni+1 ̸= nj nj+1 para


cualesquiera ı́ndices diferentes i y j, menores que 2013. Determine la menor cantidad
de números diferentes que puede tener dicha secuencia.

24
Selectivo Cono Sur 2012

Dı́a 1

1. Sea ABC un triángulo rectángulo isósceles y M el punto medio de la hipotenusa AC.


Dentro del triángulo se traza una circunferencia que es tangente a AB en P y a BC
en Q. La recta M Q corta nuevamente a la circunferencia en el punto T . Si H es el
ortocentro del triángulo AM T , demuestre que M H = BQ.
(Jorge Tipe)

2. En un tablero de 7 × 7 cada casilla se pinta de rojo o azul de tal manera que cualquier
casilla del tablero tenga al menos dos casillas vecinas azules. Determine la menor
cantidad de casillas azules que puede haber en el tablero.
Aclaración: Dos casillas son vecinas si tienen un lado en común.

3. a) Se tiene una lista de n dı́gitos no nulos (puede haber repetidos) tales que su
suma es múltiplo de 27, demuestra que esos dı́gitos se pueden ordenar de forma
adecuada para obtener un número de n dı́gitos que es múltiplo de 27.
b) Un número formado por n dı́gitos no nulos tiene la propiedad que al reordenar
sus dı́gitos de cualquier forma se obtiene siempre un múltiplo de 27, demuestra
que la suma de los n dı́gitos de ese número es múltiplo de 27.

25
Dı́a 2

4. Sea n un entero positivo. Fernando y Julián juegan de la siguiente forma: Fernando


escribe en la pizarra una lista de n enteros positivos, luego, Julián borra algunos de
ellos (puede ocurrir que no borre ninguno, pero no puede borrar todos los números) y a
cada número que queda le coloca un signo (+ o −). Si la suma de los nuevos números de
la pizarra es múltiplo de 2012, gana Julián, de lo contrario, gana Fernando. Determina
para cada valor de n quién tiene la estrategia ganadora.
Aclaración: El múltiplo de 2012 no necesariamente es positivo.

5. Una calculadora tiene dos teclas especiales:


La tecla A transforma un número x en el número 2x.
La tecla B transforma un número x en el número 2x − 1.

¿Es cierto que, si se empieza con cualquier entero positivo, es posible apretar una se-
cuencia de teclas especiales de tal forma que se obtenga al final la quinta potencia de
un número entero?

6. En un triángulo acutángulo ABC se trazan las alturas AP y BQ, y M es el punto


medio del lado AB. Si la circunferencia circunscrita al triángulo BM P es tangente al
lado AC, demuestre que la circunferencia circunscrita al triángulo AM Q es tangente
a la prolongación del lado BC.

26
Dı́a 3

7. a) Demuestre que el número 2012 no se puede expresar como la suma de los cubos
de tres números enteros.
b) Sean a y b números enteros tales que a2 − 4b es el cuadrado de un número entero,
demuestre que el número 3ab se puede expresar como la suma de los cubos de tres
números enteros.
(Jorge Tipe)

8. Se tiene un conjunto C de n circunferencias en el plano y se considera el conjunto X


de todas las rectas del plano que son tangentes al menos a dos circunferencias de C.
Se sabe que existe un polı́gono regular de 2012 lados tal que cada uno de sus lados
está incluido en alguna recta de X , determine el menor valor de n para el cual esta
situación es posible.

9. Un tablero de n×n es llamado binario si en cada casilla está escrito uno de los números
0 ó 1. En un tablero binario tenemos las siguientes definiciones:

Un rectángulo de 2 × 3 es llamado ordenado si al sumar los números de cada una


de sus tres columnas se obtiene tres números de la misma paridad.
Un rectángulo de 3 × 2 es llamado ordenado si al sumar los números de cada una
de sus tres filas se obtiene tres números de la misma paridad.

Sea A(n) la cantidad de tableros binarios de n × n que no contienen ningún rectángulo


ordenado. Sea B(n) la cantidad de tableros binarios de n × n en los que no hay dos
casillas con un lado en común que contengan ambas al número 1.
Para cada entero n ≥ 2, calcule el valor del cociente:

A(n + 1)
.
B(n)

(John Cuya)

27
Selectivo Cono Sur 2011

Dı́a 1

1. Halle todos los enteros positivos n para los cuales se cumple que:
m.c.d.(n, 1) + m.c.d.(n, 2) + · · · + m.c.d.(n, n) = 3n − 3.

Aclaración: El número m.c.d.(a, b) denota al máximo común divisor de los enteros po-
sitivos a y b.
(Sergio Vera)

2. En un torneo participaron n equipos de fútbol. Cada uno de los n equipos jugó exac-
tamente un partido contra cada uno de los otros equipos. Algunos partidos terminaron
en empate. Sucedió que cada equipo ganó exactamente tres partidos y además, no hay
tres equipos A, B, C tales que A ganó a B, B ganó a C y C ganó a A. Determine
todos los posibles valores de n.
(Jorge Tipe)

3. Considere 15 puntos en el plano, cada uno de ellos es pintado de rojo, azul o verde, de
tal manera que se cumplan las siguientes condiciones:

La suma de todas las distancias entre los puntos rojos y los azules es 51.
La suma de todas las distancias entre los puntos rojos y los verdes es 39.
La suma de todas las distancias entre los puntos azules y los verdes es 1.

Determine cuántos puntos hay de cada color (analice todas las posibilidades).

28
Dı́a 2

4. Sean M y N los puntos medios de los lados AB y AC de un triángulo ABC y G


su baricentro. Si las circunferencias circunscritas a los triángulos AM N y BGC son
tangentes exteriores, ¿es posible que el triángulo ABC sea escaleno?
(Jorge Tipe)

5. Sea n ≥ 3 un número entero. En cada una de las casillas de un tablero de n × n se


escribe un 0 o un 1 de tal manera que la suma de los números de cada subtablero 2 × 2
y de cada subtablero 3 × 3 es un número par, ¿de cuántas formas se puede hacer eso?
(Jonathan Farfán)

6. Determine todos los enteros positivos a para los cuales existen los enteros no negativos
m, n, k tales que al escribir la representación decimal de an a la izquierda de la repre-
sentación decimal de am (sin dejar espacio) obtenemos la representación decimal de
ak .
Ejemplo: Si escribimos la representación decimal de 62 a la izquierda de la representa-
ción decimal de 63 obtenemos 36216.

29
Dı́a 3

7. Un entero positivo es llamado digital si dicho número es igual al producto de los dı́gitos
de algún entero positivo. Por ejemplo, 28 es digital porque es igual al producto de los
dı́gitos del número 147.
Sean n1 , n2 , . . . , nk números digitales diferentes, demuestre que
1 1 1 35
+ + ··· + < .
n1 n2 nk 8
(Jorge Tipe)

8. Sea ABCD un cuadrilátero inscrito en una circunferencia de centro O tal que BC y


AD no son paralelos. Sea P el punto de intersección de las diagonales del cuadrilátero.
Los rayos AB y DC se intersectan en E. Una circunferencia de centro I inscrita en el
triángulo EBC es tangente al lado BC en T1 . La circunferencia ex-inscrita al triángulo
EAD, relativa a AD, es tangente a AD en T2 y tiene centro J. Las rectas IT1 y JT2
se intersectan en Q. Pruebe que O, P, Q son colineales.

9. Sea n ≥ 3 un entero impar. Cada una de las casillas de un tablero de n × n ha sido


coloreada de blanco o gris. Decimos que una secuencia de cuadrados C1 , C2 , . . . , Cm es
un camino si se cumplen las siguientes condiciones:

Los cuadrados C1 , C2 , . . . , Cm tienen el mismo color.


Los cuadrados Ci y Ci+1 comparten un lado para todo i ∈ {1, 2, . . . , m − 1}.
No hay otros dos cuadrados en la secuencia que comparten un lado.

Suponga que los cuadrados blancos forman un camino, y que los cuadrados grises
también forman un camino, demuestre que uno de esos caminos empieza o termina en
el centro del tablero.
Por ejemplo, en el tablero de la izquierda el coloreo es válido, pero en los otros dos no.
En el tablero del centro los cuadrados blancos no forman un camino porque no cumplen
la tercera condición, y en el de la derecha los cuadrados negros tampoco forman un
camino porque no cumplen la segunda condición.

30
Selectivo Cono Sur 2010

Dı́a 1

1. Un entero positivo n es llamado representable, si existen enteros positivos a > b > c


tales que n = a + b + c, y además a es múltiplo de b y b es múltiplo de c. Demuestre
que el conjunto de los enteros positivos que no son representables es finito y determine
el mayor elemento de ese conjunto.
(Jorge Tipe)

2. Sean a y b reales positivos. Determine, en función de a y b, el menor número real r que


tiene la siguiente propiedad: Es posible cubrir un rectángulo de lados a y b, con dos
discos circulares de radio r.
(Jorge Tipe)

3. Se tiene un tablero de 8 × 8 y muchas fichas de 1 × 2 y 1 × 3. Pablo debe colocar sobre


el tablero solamente fichas de 1 × 2, sin superponerse, de tal manera que sea imposible
colocar una ficha de 1 × 3 sobre las casillas descubiertas del tablero. ¿Cuál es la menor
cantidad de fichas de 1 × 2 que puede colocar Pablo?
Aclaración: Las fichas de 1 × 2 y 1 × 3 pueden estar en posición horizontal o vertical.
Cada ficha de 1 × 2 cubre exactamente dos casillas del tablero, y cada ficha de 1 × 3
cubre exactamente tres casillas del tablero.
(John Cuya)

31
Dı́a 2

4. Carlos y Daniel juegan sobre un tablero de 25 × 80, que inicialmente tiene todas sus
casillas blancas, de la siguiente forma: En su turno cada jugador elige del tablero una
región cuadrada formada solamente por casillas blancas y las pinta de negro. Carlos
inicia el juego y luego se van alternando los turnos. Si gana el jugador que pinta de
color negro la última casilla blanca del tablero, determine si hay una estrategia gana-
dora para alguno de los dos jugadores e indı́quela.

5. Sea ABC un triángulo acutángulo. En los lados AC y AB, se ubican los puntos M y N ,
respectivamente. Sean P el punto de intersección de los segmentos BM y CN , y Q un
punto en el interior del cuadrilátero AN P M tal que ∠BQC = 90 y ∠BQP = ∠BM Q.
Si el cuadrilátero AN P M es cı́clico, pruebe que ∠QN C = ∠P QC.

6. Para cada entero positivo n, sea f (n) el menor entero mayor que n para el cual existe
un conjunto M , formado por enteros positivos, que tiene las siguientes propiedades:

El menor elemento de M es n.
El mayor elemento de M es f (n).
El producto de todos los elementos de M es un cuadrado perfecto.

a) Calcule f (2010).

b) Pruebe que existen infinitos enteros positivos n para los cuales f (n) ≤ n + 2n.

(Jorge Tipe)

32
Selectivo Cono Sur 2009

Dı́a 1

1. Decimos que una sucesión formada por enteros positivos es olı́mpica si cumple las
siguientes dos condiciones:

Cada entero positivo aparece exactamente una vez en la sucesión.


Siempre que se suman tres términos consecutivos de la sucesión se obtiene un
número que no es un cuadrado perfecto.

Pruebe que existe una sucesión olı́mpica cuyo primer término es 2009.
(Jorge Tipe)

2. Considere una región poligonal regular de n lados (n ≥ 3). Pruebe que es posible
dividir dicha región en n regiones poligonales de igual área, de tal forma que cada una
de ellas tenga n lados.
Aclaración: Las regiones poligonales no son necesariamente convexas.
(Jorge Tipe)

3. En la pizarra están escritos los números


00, 01, 02, 03, 04, . . . , 96, 97, 98, 99

y se eliminan algunos de ellos por etapas. En cada etapa se eliminan exactamente 4


números de la forma

a(b − 1), a(b + 1), (a + 1)b, (a − 1)b

que no hayan sido eliminados antes, tales que 1 ≤ a ≤ 8 y 1 ≤ b ≤ 8. ¿Cuál es la mı́ni-


ma cantidad de números que pueden quedar escritos en la pizarra, luego de algunas
etapas?
(Israel Dı́az)

33
Dı́a 2

4. Para cada número natural k, sea S(k) la suma de las cifras de k en el sistema decimal,
por ejemplo, S(2009) = 2 + 0 + 0 + 9 = 11. Halle todos los números naturales n para
los cuales existen cuatro números naturales a < b < c < d, tales que

S(a) = S(b) = S(c) = S(d) = S(a + b + c + d) = n.

(Jorge Tipe)

5. Sea ABC un triángulo acutángulo, se ubican los puntos D y E en los segmentos BC


y AD, repectivamente, de tal forma que
AE CD
= .
ED DB
Sea F el pie de la perpendicular trazada desde D a la recta BE. Suponga que F perte-
nece al segmento BE y que el cuadrilátero AF DC es cı́clico. Pruebe que E pertenece
a alguna de las alturas del triángulo ABC.
(Jorge Tipe)

6. Sean P1 , P2 , P3 , . . . , P10 puntos en el espacio, algunos de ellos están unidos por segmen-
tos que no se intersectan. Un escarabajo que está en el punto P1 se puede trasladar al
punto P10 pasando por algunos de los segmentos.
Pruebe que al menos una de las dos siguientes proposiciones es verdadera:

i) El escarabajo puede ir de P1 a P10 pasando como máximo por dos puntos del
conjunto {P2 , P3 , . . . , P9 }.
ii) Existen dos puntos Pi y Pj (2 ≤ i < j ≤ 9) tales que cualquier camino del
escarabajo que une P1 con P10 pasa por el punto Pi o por el punto Pj .

Aclaración. El escarabajo se mueve solamente sobre los segmentos.

34
Selectivo Cono Sur 2008

Dı́a 1

1. ¿Cuál es el menor grado que puede tener un polinomio P (x) con coeficientes reales si
se cumple que
P (P (1)) = 2, P (P (2)) = 3, P (P (3)) = 1 ?
(Jorge Tipe)

2. Para cada n ∈ N sea D(n) el conjunto de todos los divisores positivos de n. Hallar el
menor k (en función de n) para el cual existen números naturales

1 ≤ x1 < x2 < · · · < xk ≤ n

tales que
D(x1 ) ∪ D(x2 ) ∪ · · · ∪ D(xk ) = {1, 2, . . . , n}.
(Jorge Tipe)

3. Dado un triángulo ABC, sean P y Q puntos sobre los lados AB y AC, respectivamen-
te, tales que P Q es paralelo a BC. Sean M el punto medio de BC y X el pie de la
altura trazada desde Q hacia P M . Probar que ∠AXQ = ∠QXC.
(John Cuya)

4. Encontrar todas las parejas (m, n) de enteros positivos para los cuales un tablero de
m × n puede ser cubierto, sin superposiciones, ni huecos, con fichas de la forma

(John Cuya)

35
Dı́a 2

5. Un cuadrado de lado 9 ha sido dividido en 81 cuadraditos de lado 1. Consideremos


los 100 puntos que son vértices de esos cuadraditos. Si pintamos k de esos vértices de
color rojo, ¿cuál es el mayor valor√posible de k si queremos que no haya 2 puntos rojos
cuya distancia sea de la forma m 2, donde m es un entero positivo?
(Claudio Espinoza)

6. En un concurso que consiste de dos exámenes participan N personas. Luego de la


corrección de las pruebas se elabora tres listas de la siguiente forma:

En la lista 1 aparecen las notas del primer examen.


En la lista 2 aparecen las notas del segundo examen.
En la lista 3 aparecen las sumas de las notas que cada concursante obtuvo en los
2 exámenes.

Los números escritos en las tres listas son todos distintos, y en cada lista están orde-
nados de mayor a menor.
Un concursante se dice clasificado si su nota pertenece al medio superior de la lista 1
o al medio superior de la lista 2, y además pertenece al tercio superior de la lista 3.
¿Cuál es el menor número de clasificados que puede haber?
N 
Aclaración.  el medio superior están las 2 mayores notas. En el tercio superior
 N En
están las 3 mayores notas.
(Sergio Vera)

7. Halle todos los números primos p ≥ 3 para los cuales el número 1 + k(p − 1) es primo,
p−1
para cualquier entero positivo k ≤ .
2

36
Dı́a 3

8. Se tiene el hexágono convexo ABCDEF tal que ∠F AB = ∠CDE = 90 y el cua-


drilátero BCEF es circunscriptible. Pruebe que AD ≤ BC + F E.
(John Cuya)

9. Emilio y Mariano juegan en un tablero 13 × 13 de la siguiente forma: Emilio escoge k


casillas del borde del tablero y pinta cada una de ellas de negro o blanco, luego Mariano
pinta cada una de las otras (169 − k) casillas de negro o blanco, si luego de que todas
las casillas están pintadas, Emilio encuentra un subtablero 2 × 2 que tenga un número
impar de casillas negras él gana, caso contrario Mariano gana. Halle el menor k para
el cual Emilio tiene estrategia ganadora.
(Jorge Tipe)

10. Para cada entero positivo n, sea S(n) la suma de las cifras de n. Sean a y b dos enteros
positivos distintos y no divisibles por 10.

a) Pruebe que existe un entero positivo c tal que S(c · a) ̸= S(c · b).
b) Pruebe que existe un entero positivo c tal que S(c · a) > S(c · b).

(Jonathan Farfán)

37
Selectivo Cono Sur 2007

1. Dado un cuadrado ABCD, sean M, K, L y N puntos sobre los lados AB, BC, CD y
DA, respectivamente, tales que ∠M KA = ∠KAL = ∠ALN = 45◦ . Pruebe que

M K 2 + AL2 = AK 2 + LN 2 .

2. Inicialmente se tienen los números 1, 2, 3, 4 escritos alrededor de un cı́rculo (en ese


orden). Dos jugadores A y B juegan en forma alternada de la siguiente manera, co-
menzando el jugador A: A elige dos números vecinos y le suma 1 a ambos, B, a su
turno, elige dos números vecinos y los intercambia de lugar. A gana si consigue que
todos los números sean iguales. ¿Puede evitar B que A gane?

3. Encuentre todos los enteros positivos n tales que n + 1 se pueda expresar como la suma
de tres divisores positivos de n distintos entre sı́.

4. a) Pruebe que los números enteros del 1 al 16 pueden ser distribuidos en un tablero
de 4 × 4 , uno en cada casilla, de tal manera que la suma de los números escritos
en dos casillas vecinas cualesquiera sea un número primo.
b) ¿Se cumplirı́a lo mismo si en vez de los números del 1 al 16 se distribuyen los
números del 2 al 17?
Aclaración Dos casillas son vecinas si tienen un lado en común.

38
Selectivo Cono Sur 2006

1. Encontrar todos los pares de números enteros positivos tales que el último dı́gito de
su suma es 3, su diferencia es un número primo y su producto es un entero cuadrado
perfecto.

2. AA1 y BB1 son las alturas de un triángulo acutángulo no isósceles ABC. A0 y B0 son
los puntos medios de BC y CA, respectivamente. El segmento A1 B1 corta al segmento
A0 B0 en C ′ . Probar que CC ′ es perpendicular a la recta que une el ortocentro y cir-
cuncentro del triángulo ABC.

3. El conjunto M = {1, 2, 3, . . . , 29, 30} se divide en k subconjuntos de tal manera que si


a + b = n2 , ( a, b ∈ M , a ̸= b, n es un número entero), entonces a y b pertenecen a
diferentes subconjuntos. Determinar el menor valor de k.

4. Todas las casillas de un tablero cuadriculado de (n + 1) × (n − 1) casillas son pintadas


con tres colores de modo que, para cada dos columnas distintas cualesquiera y cada dos
filas distintas cualesquiera, las cuatro casillas en sus intersecciones no sean pintadas
todas del mismo color. Encontrar el mayor valor posible de n.

39
Selectivo Cono Sur 2005

1. Los enteros positivos 1, 2, 3, ..., se escriben en las casillas de la cuadrı́cula siguiente,


uno en cada casilla, de la forma siguiente:

fila n

... 15

fila 4 10 14

fila 3 6 9 13

fila 2 3 5 8 12

fila 1 1 2 4 7 11
col. col. col. col. col ... col.
1 2 3 4 5 m

Encuentre un polinomio P (x, y) , tal que para cualesquiera enteros positivos m, n el


número escrito en la casilla ubicada en la columna m y fila n sea P (m, n).

2. Sobre veinte puntos en una circunferencia se ubican veinte fichas. Dos jugadores, en
forma alternada, retiran tres fichas cualesquiera en cada jugada, hasta que solamente
queden dos fichas. Si las dos fichas que quedan eran adyacentes en la ubicación inicial,
el jugador que comienza gana; en caso contrario, el otro jugador gana. Analizar cuál
de los dos jugadores tiene una estrategia ganadora.

3. Sea D el punto medio del lado BC de un triángulo dado ABC. Sean M un punto
del lado BC tal que ∠BAM = ∠DAC, L el segundo punto de intersección del cir-
cuncı́rculo del triángulo CAM con el lado AB y K el segundo punto de intersección del
circuncı́rculo del triángulo BAM con el lado AC. Pruebe que KL y BC son paralelos.

1 a2n
4. Sea (an ) la sucesión definida por a1 = y an+1 = 2 , para n ≥ 1. Pruebe
2 an − an + 1
que para cualquier entero positivo n se cumple que:
n
X
ak < 1.
k=1

40
Selectivo Cono Sur 2004

1. En una loterı́a especial, al comprar un ticket de loterı́a, un jugador debe elegir 6


números de 36 posibles. Al momento del sorteo se seleccionan, al azar, 6 números de
los 36 disponibles y un ticket es ganador si ninguno de sus números fue seleccionado
en el sorteo.
a) Probar que es posible comprar 9 tickes de tal manera que al menos uno de ellos
será ganador.
b) Probar que no es posible comprar 8 tickets de tal manera que al menos uno de
ellos será ganador.

2. Dos piratas encontraron un cofre conteniendo monedas de valores a1 < a2 < . . . < a2003
(hay suficiente cantidad de monedas de cada valor). El primer pirata forma todos los
posibles conjuntos de monedas de distintos valores que contienen un número impar de
monedas, y toma de cada conjunto la moneda de mayor valor. El segundo pirata forma
todos los posibles conjuntos de monedas de distintos valores que contienen un número
par de monedas y toma de cada conjunto la moneda de mayor valor. ¿Cuál de ellos se
lleva mayor cantidad de dinero y cuánto más?

3. Los números reales α y β satisfacen:


α3 − 3α2 + 5α − 17 = 0,
β 3 − 3β 2 + 5β + 11 = 0
Encontrar α + β.

4. En el interior de un triángulo ABC se construyen cuatro circunferencias K1 , K2 , K3 y


K4 , del mismo radio, tales que K1 , K2 y K3 son tangentes a dos lados del triángulo y
a K4 , como se muestra en la figura:

K3

K4

K2
K1
A B

Probar que el centro de K4 está ubicado sobre la recta que pasa por el incentro y el
circuncentro del triángulo.

41
Selectivo Cono Sur 2003

1. Determinar todos los números reales a tales que la ecuación x8 + ax4 + 1 = 0 tenga
cuatro raı́ces reales que formen una progresión aritmética.

2. Sean p y n enteros positivos tales que p es primo y 1 + np es un cuadrado perfecto.


Probar que el número n + 1 puede ser expresado como la suma de p cuadrados perfec-
tos, donde algunos de ellos pueden ser iguales.

3. Sean M y N puntos sobre el lado BC de un triángulo ABC tales que BM = CN


(M se encuentra entre B y N ). Los puntos P y Q se encuentran respectivamente so-
bre AN y AM , de modo que ∠P M C = ∠M AB y ∠QN B = ∠N AC. Probar que
∠QBC = ∠P CB.

4. Ocho fichas se encuentran sobre un tablero de 8×8 de tal modo que ningún par de ellas
están en una misma fila ni en una misma columna. Probar que, entre las distancias entre
cada par de fichas, podemos encontrar dos de ellas que son iguales (la distancia entre
dos fichas es la distancia entre los centros de las casillas en las que ellas se encuentran).

42
Selectivo Cono Sur 2002

1. Sean n un número entero positivo y a1 , a2 , . . . , an números reales positivos tales que


a1 < a2 < · · · < an . Pruebe que:

12 22 32 n2 n n−1 n−2 1
+ + + ··· + ≤ + + + ··· + .
a1 a2 a3 an a1 a2 − a1 a3 − a2 an − an−1

2. Encuentre todos los pares de números enteros (x, y) que satisfacen la ecuación
1 + x2 y = x2 + 2xy + 2x + y.

3. Sean AD, BE y CF las bisectrices interiores del triángulo ABC. Demostrar que si uno
de los ángulos ∠ADF , ∠ADE, ∠BED, ∠BEF , ∠CF E, ∠CF D mide 30◦ entonces al
menos uno más de estos ángulos mide 30◦ .

4. Determine el menor entero positivo n ≥ 4 para el cual existe un conjunto de n niños


tal que:

En el conjunto no existe un grupo de 4 niños para el cual cada dos de ellos son
amigos.
Para cualquier elección de k niños del conjunto (k ≥ 1), entre los cuales no hay
amigos, existe, entre los restantes n − k niños, un grupo de 3 niños para el cual
cada dos niños son amigos.

43
Exámenes Selectivos para la IMO

Comisión de Olimpiadas de la Sociedad Matemática Peruana

Edición: Jorge Tipe


Versión: diciembre 2018
Prólogo

La Olimpiada Internacional de Matemática (IMO, por sus siglas en inglés) es la olimpiada


matemática más importante, y participan actualmente más de 100 paı́ses. El Perú participa
desde el año 1987, con algunas interrupciones en los años 90, pero de forma continua desde
el año 1997. Hasta la fecha, año 2018, el Perú ha obtenido 5 medallas de oro, 29 de plata
y 47 de bronce. La mejor ubicación de nuestro paı́s ha sido el lugar 16, esta ubicación se
consiguió en los años 2012 y 2015.
En el Perú, la Comisión de Olimpiadas de la Sociedad Matemática Peruana está a cargo de la
selección de los alumnos desde que empezó sus actividades, hace más de 30 años, y con este
fin se toman exámenes selectivos varios meses antes de la realización de cada olimpiada. En
el caso de los exámenes selectivos para la IMO, desde el año 2014 se adoptó el sistema de un
examen preselectivo y varios exámenes selectivos adicionales en los que solamente participan
los alumnos preseleccionados.
En este archivo podrán encontrar todos los problemas usados en el periodo 2007-2018; si
encuentran un error, tienen una sugerencia para aclarar la redacción de un problema, o si
tienen cualquier otra consulta con respecto a este archivo, me pueden escribir a mi correo
personal jorgetipe@gmail.com por lo cual estaré muy agradecido. Iré actualizando este
archivo con el paso del tiempo. Por ejemplo, si consigo exámenes de años anteriores.

Jorge Tipe Villanueva

Comisión de Olimpiadas
de la Sociedad Matemática Peruana

1
Selectivo IMO 2018

Pre-selectivo

1. Un rectángulo R de lados enteros impares es dividido en rectángulos menores de lados


enteros. Probar que entre todos los rectángulos menores, existe al menos uno cuyas
distancias a los cuatro lados de R son todas impares o todas pares.

2. Sean a1 , a2 , . . . , an , k y M enteros positivos tales que


1 1 1
+ + ··· + =k y a1 a2 . . . an = M.
a1 a2 an
Si M > 1, probar que el polinomio

P (x) = M (x + 1)k − (x + a1 )(x + a2 ) . . . (x + an )

no tiene ninguna raı́z real positiva.

3. Sea ABCDE un pentágono convexo tal que AB = BC = CD, ∠EAB = ∠BCD y


∠EDC = ∠CBA. Probar que la recta perpendicular desde E hacia BC y los segmen-
tos AC y BD son concurrentes.

4. Encontrar todos los pares (p, q) de números primos, con p > q, para los cuales el
número
(p + q)p+q (p − q)p−q − 1
(p + q)p−q (p − q)p+q − 1
es un entero.

2
Dı́a 1

5. Sea d un entero positivo. La sucesión a1 , a2 , a3 , . . . de enteros positivos es definida por


 
an
a1 = 1 y an+1 =n + d para n = 1, 2, 3, . . .
n

Pruebe que existe un entero positivo N de modo que los términos aN , aN +1 , aN +2 , . . .


forman una progresión aritmética.
Nota: Si x es un número real, bxc denota al mayor entero que es menor o igual que x.

6. Sea n un entero positivo. Un camaleón es una sucesión de 3n letras con exactamente n


letras a, n letras b y n letras c. Un cambio en un camaleón consiste en seleccionar dos
de sus letras adyacentes e intercambiarlas. Ana selecciona un camaleón y a continua-
ción Beto también selecciona un camaleón. Ana debe entonces partir de su camaleón y
mediante cambios obtener el camaleón de Beto. Pruebe que sin importar cuál camaleón
seleccione Ana, Beto siempre puede seleccionar su camaleón de modo que Ana deba
3n2
realizar al menos cambios para cumplir su objetivo.
2

7. Sean ABC, con AC > AB, un triángulo acutángulo con circuncı́rculo Γ y M el punto
medio del lado BC. Sea N un punto interior al triángulo ABC. Sean D y E respec-
tivamente los pies de las perpendiculares trazadas desde N a AB y a AC. Suponga
que DE es perpendicular a AM . El circuncı́rculo del triángulo ADE corta a Γ en L
(L 6= A), las rectas AL y DE se cortan en K y la recta AN corta a Γ en F (F 6= A).
Pruebe que si N es el punto medio del segmento AF entonces KA = KF .

3
Dı́a 2

8. Se quieren pintar de rojo algunas aristas de un dodecaedro regular de manera que cada
cara tenga una cantidad par de aristas pintadas (que puede ser cero). Determine de
cuántas maneras se puede realizar esta coloración.
Nota: Un dodecaedro regular tiene doce caras pentagonales y en cada vértice concu-
rren tres aristas. Las aristas del dodecaedro son todas distintas para el propósito de
las coloraciones. De esa forma, dos coloraciones son iguales sólo si las aristas pintadas
son las mismas.

9. Sea a1 , a2 , a3 , . . . una sucesión de números reales tal que


an = − máx (ai + aj ) para todo entero n > 2017.
i+j=n

Pruebe que existe un número real M tal que |an | ≤ M para todo entero positivo n.

10. Para cada entero positivo m > 1, sea P (m) el producto de todos los números primos
que dividen a m. Se define la sucesión a1 , a2 , a3 , . . . de la siguiente manera:

a1 > 1 es un entero positivo arbitrario,


an+1 = an + P (an ) para cada entero positivo n.

Pruebe que existen enteros positivos j y k tales que aj es el producto de los k primeros
números primos.

4
Selectivo IMO 2017

Pre-selectivo

1. Halle todas las funciones f : R → R tales que


f (xy − 1) + f (x)f (y) = 2xy − 1,

para todo x, y ∈ R.

2. Sea n ≥ 3 un entero. Mario dibuja n rectas en el plano, tales que no haya dos para-
lelas. Por cada triángulo equilátero formado por tres de esas rectas, Mario recibe tres
monedas. Por cada triángulo isósceles y no equilátero (a la vez) formado por tres de
esas rectas, Mario recibe una moneda. ¿Como máximo cuántas monedas puede recibir
Mario?

3. La circunferencia inscrita del triángulo ABC es tangente a los lados BC, AC y AB


en los puntos D, E y F , respectivamente. Sea M el punto medio de EF . La circunfe-
rencia circunscrita del triángulo DM F intersecta a la recta AB en L; la circunferencia
circunscrita del triángulo DM E intersecta a la recta AC en K. Pruebe que la circun-
ferencia circunscrita del triángulo AKL es tangente a la recta BC.

4. El producto
1 × 2 × 3 × ··· × n
está escrito en la pizarra. ¿Para qué enteros n ≥ 2, podemos agregar signos de excla-
mación a algunos factores para convertirlos en factoriales, de tal forma que el producto
final sea un cuadrado perfecto?

5
Dı́a 1

5. Halle el menor número real C que tiene la siguiente propiedad: Para cualesquiera
números reales positivos a1 , a2 , a3 , a4 y a5 (no necesariamente distintos), existen ı́ndices
distintos i, j, k y ` tales que
ai ak
− ≤ C.
aj a`

6. Sea n un entero positivo coprimo con 6. Pintamos los vértices de un polı́gono regular
de n lados con tres colores de tal forma que haya un número impar de vértices de cada
color. Pruebe que existe un triángulo isósceles que tiene sus tres vértices de colores
distintos.

7. Sea I el incentro de un triángulo no equilátero ABC, IA el A-excentro, IA0 el simétrico


de IA con respecto a la recta BC, y `A la reflexión de la recta AIA0 en AI. Se definen
análogamente los puntos IB , IB0 y la recta `B . Sea P el punto de intersección de `A y
`B .

a) Pruebe que P pertenece a la recta OI, donde O es el circuncentro del triángulo


ABC.
b) Si una de las rectas que pasa por P y es tangente a la circunferencia inscrita
del triángulo ABC intersecta al circuncı́rculo en los puntos X y Y , pruebe que
∠XIY = 120◦ .

6
Dı́a 2

8. El lı́der de un equipo de la IMO escoge enteros positivos n y k, con n > k, y los anuncia
al tutor y a un alumno. En secreto, el lı́der le dice al tutor una cadena binaria de n
dı́gitos, y el tutor escribe todas las cadenas binarias de n dı́gitos que difieren de la
cadena del lı́der en exactamente k posiciones. (Por ejemplo, si n = 3 y k = 1, y el lı́der
escoge 101, el tutor escribe 001, 111 y 100.) El alumno tiene permitido ver todas las
cadenas escritas por el tutor y adivinar la cadena del lı́der. ¿Cuál es el mı́nimo número
de intentos (en función de n y k) necesarios para que el alumno asegure encontrar la
cadena correcta?

9. Sea ABCD un cuadrilátero cı́clico, ω su circuncı́rculo y M el punto medio del arco AB


de ω que no contiene a los vértices C y D. La recta que pasa por M y por el punto de
intersección de los segmentos AC y BD, intersecta nuevamente a ω en N . Sean P y Q
puntos en el segmento CD tal que ∠AQD = ∠DAP y ∠BP C = ∠CBQ. Pruebe que
el circuncı́rculo de N P Q y ω son tangentes entre sı́.

10. Sean P (n) y Q(n) dos polinomios (no constantes) cuyos coeficientes son enteros no
negativos. Para cada entero positivo n, definimos xn = 2016P (n) + Q(n). Pruebe que
existen infinitos primos p para los cuales existe un entero positivo m, libre de cuadrados,
tal que p | xm .
Aclaración. Un entero positivo es libre de cuadrados si no es divisible por el cuadrado
de cualquier número primo.

7
Dı́a 3

11. Sea ABC un triángulo acutángulo escaleno de circuncı́rculo Γ y ortocentro H. Sean


A1 , B1 , C1 los segundos puntos de intersección de las rectas AH, BH, CH con Γ,
respectivamente. Las rectas que pasan por A1 , B1 , C1 y son paralelas a BC, CA, AB
intersectan nuevamente a Γ en A2 , B2 , C2 , respectivamente. Sea M el punto de inter-
sección de AC2 y BC1 ; N el punto de intersección de BA2 y CA1 ; y P el punto de
intersección de CB2 y AB1 . Pruebe que ∠M N B = ∠AM P .

12. Sea a un entero positivo que no es un cuadrado perfecto. Sea A el conjunto de todos
los enteros positivos k tales que

x2 − a
k= (1)
x2 − y 2

para algunos enteros x y y con x > a. Sea B el conjunto de los √
enteros positivos k para
los cuales se cumple (1) para algunos enteros x y y con 0 ≤ x < a. Pruebe que A = B.

13. Sea R el conjunto de los números reales. Halle todas las funciones f : R → R tales que
f (0) 6= 0 y

f (x + y)2 = 2f (x)f (y) + máx{f (x2 ) + f (y 2 ), f (x2 + y 2 )},

para todos los números reales x, y.

8
Dı́a 4

14. Para cada entero positivo k, sea S(k) la suma de los dı́gitos de k. Halle todos los
polinomios P (x), de coeficientes enteros, tales que para todo entero n ≥ 2016, se
cumple que P (n) es positivo y

S(P (n)) = P (S(n)).

a
15. a) Pruebe que para todo entero positivo n, existe una fracción , donde a y b son
√ √ √ b
a
enteros tales que 0 < b ≤ n+1 y n≤ b
≤ n + 1.
b) Pruebe que existen infinitos enteros positivos n para los cuales no existe una frac-
a √ √ √
ción , donde a y b son enteros tales que 0 < b ≤ n y n ≤ ab ≤ n + 1.
b

16. Sean n y k enteros positivos. Un grafo simple G no contiene ningún ciclo cuya longi-
tud sea un número impar mayor que 1 y menor que 2k + 1. Si G tiene como máximo
n + (k−1)(n−1)(n+2)
2
vértices, pruebe que los vértices de G se pueden pintar con n colores
de tal forma que cualquier arista de G tenga sus extremos de colores diferentes.

9
Selectivo IMO 2016

Pre-selectivo

1. Los reales positivos a, b, c satisfacen la ecuación ab + bc + ca = 1. Pruebe que



r r r
1 1 1 √ √
a+ + b+ + c+ ≥ 2( a + b + c).
a b c

2. Determine cuántas secuencias de 100 enteros positivos satisfacen las dos condiciones
siguientes:

Al menos un término de la secuencia es igual a 4 o 5.


Dos términos adyacentes cualesquiera difieren como máximo en 2.

3. Sea ABCD un cuadrilátero convexo tal que AD y BC no son paralelos. Sean M y N


los puntos medios de los lados AD y BC, respectivamente. El segmento M N corta a
AC y BD en K y L, respectivamente. Pruebe que uno de los puntos de intersección de
las circunferencias circunscritas de los triángulos AKM y BN L pertenece a la recta
AB.

4. Sea N el conjunto de los enteros positivos. Halle todas las funciones f : N → N con
f (1) = 2 y tales que máx(f (m) + f (n), m + n) divide a mı́n(2m + 2n, f (m + n) + 1)
para todo m, n enteros positivos.

10
Dı́a 1

5. Sea ABC un triángulo acutángulo de ortocentro H. Sea G un punto tal que ABGH es
un paralelogramo. Sea I el punto de la recta GH tal que AC divide al segmento HI en
dos partes iguales. Suponga que la recta AC intersecta a la circunferencia circunscrita
del triángulo GCI en C y J. Pruebe que IJ = AH.

6. Determine todas las funciones f : Z → Z tales que


f (x − f (y)) = f (f (x)) − f (y) − 1,

para todos los x, y ∈ Z.

7. Sea S un conjunto no vacı́o de enteros positivos. Decimos que un entero positivo n


es limpio si tiene exactamente una representación como suma de una cantidad impar
de elementos distintos de S. Pruebe que existen infinitos enteros positivos que no son
limpios.

11
Dı́a 2

8. Suponga que la sucesión a1 , a2 , . . . de reales positivos cumple que


kak
ak+1 ≥
a2k + (k − 1)

para todos los enteros positivos k. Pruebe que a1 + a2 + · · · + an ≥ n para todo entero
n ≥ 2.

9. Sea Z>0 el conjunto de los enteros positivos. Para cada entero positivo k, una función
f : Z>0 → Z>0 es llamada k-buena si mcd(f (m) + n, f (n) + m) ≤ k para todos los
enteros positivos m y n, con m 6= n.

a) Pruebe que no existe una función 1-buena.


b) ¿Existe una función 2-buena?

10. Sea ABC un triángulo con CA 6= CB. Sean D, F, G los puntos medios de los lados
AB, AC, BC, respectivamente. Una circunferencia Γ que pasa por C y es tangente a
AB en D intersecta a los segmentos AF y BG en H e I, respectivamente. Los puntos
H 0 e I 0 son los simétricos de H e I con respecto a F y G, respectivamente. La recta
H 0 I 0 intersecta a CD y F G en Q y M , respectivamente. La recta CM intersecta a Γ
nuevamente en P . Pruebe que CQ = QP .

12
Dı́a 3

11. Sea n > 2 un entero. Un niño tiene n2 caramelos, que están distribuidos en n cajas.
Una operación consiste en escoger dos cajas que contengan en conjunto un número par
de caramelos y redistribuir los caramelos de esas cajas para que ambas contengan la
misma cantidad de caramelos. Determine todos los valores de n para los cuales el niño,
luego de algunas operaciones, puede conseguir que cada caja contenga n caramelos, sin
importar cuál sea la distribución inicial de los caramelos.

12. Sea ABC un triángulo con ∠C = 90◦ , y sea H el pie de la altura trazada desde C. Un
punto D es escogido dentro del triángulo CBH tal que CH pasa por el punto medio de
AD. Sea P el punto de intersección de las rectas BD y CH. Sea ω la semicircunferencia
de diámetro BD que intersecta al segmento CB en un punto interior. Una recta que
pasa por P es tangente a ω en Q. Pruebe que las rectas CQ y AD se intersectan en
un punto que pertenece a ω.

13. Sea Z>0 el conjunto de los enteros positivos. Considere una función f : Z>0 → Z>0 .
Para cualesquiera m, n ∈ Z>0 escribimos f n (m) = f (f (. . . f (m) . . .)). Suponga que f
| {z }
n
tiene las siguientes propiedades:
f n (m) − m
(i) Si m, n ∈ Z>0 entonces ∈ Z>0 ;
n
(ii) El complemento del conjunto {f (n) | n ∈ Z>0 } con respecto a Z>0 es finito.

Pruebe que la sucesión f (1) − 1, f (2) − 2, f (3) − 3, . . . es periódica.

13
Dı́a 4

14. Determine todos los enteros positivos M para los cuales la sucesión a0 , a1 , a2 , . . . defi-
2M + 1
nida por a0 = y ak+1 = ak · bak c para k = 0, 1, 2, . . . contiene al menos un
2
término entero.

15. Sea n un entero positivo. Los jugadores A y B realizan un juego en el que alternan
turnos, y en cada turno un jugador escoge un entero positivo k ≤ n. Las reglas del
juego son:

(i) Un jugador no puede escoger un número que haya sido escogido por alguno de los
jugadores en un turno anterior.
(ii) Un jugador no puede escoger un número adyacente a alguno de los números que
ese mismo jugador haya escogido en un turno anterior.
(iii) El juego es un empate si todos los números han sido escogidos; de otra forma, el
jugador que no puede escoger un número en su turno pierde el juego.

El jugador A realiza el primer turno. Determine el resultado del juego, asumiendo que
los dos jugadores usan estrategias óptimas.

Aclaración: Un número entero a es adyacente de b si |a − b| = 1.

16. Halle todas las parejas (m, n) de enteros positivos que tienen la siguiente propiedad:
Para todo polinomio P (x) de coeficientes reales y grado m, existe un polinomio Q(x)
de coeficientes reales y grado n tal que Q(P (x)) es divisible por Q(x).

14
Selectivo IMO 2015

Pre-selectivo

1. Halle todos los enteros positivos n para los cuales existen números reales x1 , x2 , . . . , xn
que satisfacen las siguientes condiciones (a la vez):

(i) −1 < xi < 1 para todo 1 ≤ i ≤ n.


(ii) x1 + x2 + · · · + xn = 0.
p p p
(iii) 1 − x21 + 1 − x22 + · · · + 1 − x2n = 1.

2. Ana escogió algunas casillas de un tablero de 50 × 50 y colocó una ficha en cada una
de ellas. Pruebe que Beto siempre puede escoger a lo más 99 casillas vacı́as y colocar
una ficha en cada una de ellas de tal forma que cada fila y cada columna del tablero
contenga un número par de fichas.

3. Sean M el punto medio del arco BAC de la circunferencia circunscrita del triángu-
lo ABC, I el incentro del triángulo ABC y L un punto del lado BC tal que AL es
bisectriz. La recta M I vuelve a cortar a la circunferencia circunscrita en el punto K.
La circunferencia circunscrita del triángulo AKL vuelve a cortar a la recta BC en P .
Pruebe que ∠AIP = 90◦ .

4. Sea n ≥ 2 un entero. La permutación a1 , a2 , . . . , an de los números 1, 2, . . . , n es llamada


cuadrática si ai ai+1 +1 es un cuadrado perfecto para todo 1 ≤ i ≤ n−1. La permutación
a1 , a2 , . . . , an de los números 1, 2, . . . , n es llamada cúbica si ai ai+1 + 1 es un cubo
perfecto para todo 1 ≤ i ≤ n − 1.

a) Pruebe que para infinitos valores de n existe al menos una permutación cuadrática
de los números 1, 2, . . . , n.
b) Pruebe que para ningún valor de n existe una permutación cúbica de los números
1, 2, . . . , n.

15
Dı́a 1

5. Tenemos 2m hojas de papel, con el número 1 escrito en cada una de ellas. Realizamos
la siguiente operación. En cada paso escogemos dos hojas de papel, si los números en
las dos hojas son a y b, borramos esos dos números y escribimos el número a + b en
ambas hojas. Pruebe que después de m · 2m−1 pasos, la suma de los números de todas
las hojas es al menos 4m .

6. Sea n > 1 un entero. Pruebe que infinitos términos de la sucesión (ak )k≥1 , definida por
nk
 
ak =
k

son impares.
Nota: Para un número real x, bxc denota al mayor entero que es menor o igual que x.

7. Para una secuencia x1 , x2 , . . . , xn de números reales, definimos su precio como:


máx |x1 + · · · + xi |.
1≤i≤n

Dados n números reales, Ana y Beto quieren ordenarlos en una secuencia que tenga
un precio bajo. Ana comprueba todas las posibles formas y encuentra el menor precio
A. Por otro lado, Beto escoge x1 tal que |x1 | sea lo menor posible; entre los números
que quedan, él escoge x2 tal que |x1 + x2 | sea lo menor posible, y ası́ sucesivamente. Es
decir, en el i-ésimo paso él escoge xi entre los números que quedan tal que minimice el
valor de |x1 + x2 + · · · + xi |. En cada paso, si varios números proporcionan el mismo
valor, Beto escoge uno al azar. Finalmente Beto obtiene una secuencia con precio B.
Halle la menor constante c tal que para cada entero positivo n, para cada colección de n
números reales, y para cada secuencia que Beto podrı́a obtener, los valores resultantes
satisfacen la desigualdad B ≤ cA.

16
Dı́a 2

8. Sea I el incentro del triángulo ABC. La circunferencia que pasa por I y tiene centro
en A intersecta a la circunferencia circunscrita del triángulo ABC en los puntos M y
N . Pruebe que la recta M N es tangente a la circunferencia inscrita del triángulo ABC.

9. Sea A un conjunto finito de funciones f : R → R. Se sabe que:


Si f, g ∈ A entonces f (g(x)) ∈ A.
Para todo f ∈ A existe g ∈ A tal que f (f (x) + y) = 2x + g(g(y) − x), para todos
los reales x, y.

Sea i : R → R la función identidad, es decir, i(x) = x para todo x ∈ R. Pruebe que


i ∈ A.

10. Se tiene una caja que contiene 1024 tarjetas. En cada tarjeta está escrito un conjunto
de dı́gitos decimales de tal forma que todos los conjuntos son diferentes (en particular,
una de las tarjetas está vacı́a). Dos jugadores, de forma alternada, retiran tarjetas de
la caja (una tarjeta en cada turno). Después de que la caja está vacı́a, cada jugador
revisa si puede retirar una de sus tarjetas tal que cada uno de los diez dı́gitos aparezca
un número par de veces entre las tarjetas que le quedan a ese jugador. Si un jugador
puede hacer esto pero el otro no puede, el que sı́ puede es el ganador, de otra manera
se declara un empate.
Determine todas las primeras jugadas que puede realizar el primer jugador para que él
tenga estrategia ganadora.

17
Dı́a 3

11. Sea n ≥ 2 un entero y sea An el conjunto


An = { 2n − 2k | k ∈ Z, 0 ≤ k < n }.

Determine el mayor entero positivo que no se puede expresar como la suma de uno o
más (no necesariamente distintos) elementos de An .

12. Halle el menor número real positivo α que tiene la siguiente propiedad: Si el peso de
un número finito de calabazas es 1 tonelada y el peso de cada calabaza no es mayor
que α toneladas entonces las calabazas se pueden distribuir en 50 cajas (algunas de las
cajas pueden quedar vacı́as) de tal forma que no haya más de α toneladas de calabazas
en cada caja.

13. Sea ABC un triángulo con circuncı́rculo Ω e incentro I. La recta que pasa por I y es
perpendicular a CI intersecta al segmento BC y al arco BC de Ω (que no contiene a
A) en los puntos U y V , respectivamente. La recta que pasa por U y es paralela a AI
intersecta a AV en X; y la recta que pasa por V y es paralela a AI intersecta a AB
en Y . Sean W y Z los puntos medios de AX y BC, respectivamente. Pruebe que si los
puntos I, X y Y son colineales, entonces los puntos I, W y Z también son colineales.

18
Dı́a 4

14. Sea n un entero positivo y x1 , x2 , . . . , xn reales positivos tales que


1 1 1
x1 + x2 + · · · + xn = 2
+ 2 + ··· + 2 .
x1 x 2 xn

Pruebe que para cada entero positivo k ≤ n, existen k números entre x1 , x2 , . . . , xn


cuya suma es al menos k.

15. Sea ABC un triángulo. Los puntos K, L y M pertenecen a los segmentos BC, CA y
AB, respectivamente, tales que las rectas AK, BL y CM son concurrentes. Pruebe que
es posible escoger dos de los triángulos ALM , BM K y CKL cuyos inradios tengan
suma mayor o igual que el inradio del triángulo ABC.

16. Sea c ≥ 1 un entero. Definimos la sucesión de enteros positivos (an ) haciendo a1 = c y


an+1 = a3n − 4c · a2n + 5c2 · an + c

para todo n ≥ 1. Pruebe que para cada entero n ≥ 2 existe un primo p que divide a
an pero que no divide a ninguno de los números a1 , . . . , an−1 .

19
Selectivo IMO 2014

Pre-selectivo

1. Sea R+ el conjunto de los reales positivos.


a) Construya una función f : R+ → R+ tal que

2f (x2 ) ≥ xf (x) + x, para todo x ∈ R+ .

b) Probar que si f (x) cumple la condición de la parte a) entonces f (x3 ) ≥ x2 , para


todo x ∈ R+ .

2. Sea n un entero positivo. Hay un número infinito de tarjetas y cada una tiene escrito un
número entero no negativo, de tal forma que para cada entero ` ≥ 0 hay exactamente n
tarjetas que tienen escrito el número `. Una operación consiste en escoger 100 tarjetas
de la colección infinita de cartas y luego desecharlas. Hallar el menor valor posible de n
para el cual se puede realizar una serie infinita de operaciones tal que para cada entero
positivo k se cumpla que la suma de los números de las 100 tarjetas escogidas en la
k-ésima operación sea igual a k.

3. Sea ABC un triángulo acutángulo con AB > BC inscrito en una circunferencia. La


mediatriz del lado AC corta al arco AC que contiene a B en Q. Sea M un punto del
segmento AB tal que AM + BC = M B. Probar que la circunferencia circunscrita al
triángulo BM C corta a BQ en su punto medio.

4. Un entero positivo es llamado solitario si la suma de las inversas de sus divisores


positivos (incluyendo a 1 y a sı́ mismo) es diferente de la suma de las inversas de los
divisores positivos de cualquier otro entero positivo.

a) Demostrar que cualquier número primo es solitario.


b) Probar que hay infinitos enteros positivos que no son solitarios.

20
Dı́a 1

5. Escogemos n vértices cualesquiera de un polı́gono regular de 2n ladosy los coloreamos


de rojo. Los otros n vértices son coloreados de azul. Ordenamos las n2 distancias entre
dos vértices rojos para formar una secuencia no decreciente, y hacemos lo mismo con
las n2 distancias entre dos vértices azules. Pruebe que las dos secuencias son idénticas.


6. Dado un triángulo ABC con AB > BC, sean D y E puntos de los lados AB y BC,
respectivamente, tales que DE y AC son paralelos. Consideremos la circunferencia
circunscrita del triángulo ABC. Una circunferencia que pasa por los puntos D y E es
tangente al arco AC que no contiene a B en el punto P . Sea Q la reflexión del punto
P respecto a la mediatriz de AC. Los segmentos BQ y DE se cortan en el punto X.
Pruebe que AX = XC.

7. Sea n un entero positivo. Mariano divide un rectángulo en n2 rectángulos menores


mediante n − 1 rectas verticales y n − 1 rectas horizontales, paralelas a los lados del
rectángulo mayor. En cada paso Emilio escoge uno de esos rectángulos menores y Ma-
riano le dice su área. Encuentre el menor entero positivo k para el cual siempre es
posible que Emilio realice k pasos (escogidos convenientemente) de tal forma que con
la información recibida pueda determinar el área de cada uno de los n2 rectángulos
menores.

21
Dı́a 2

8. Sean x, y, z números reales tales que


(
x2 + y 2 + z 2 + (x + y + z)2 = 9,
15
xyz ≤ 32 .

Determine el mayor valor posible de x.

9. Pruebe que para todo entero positivo n existen enteros a y b, ambos mayores que 1,
tales que a2 + 1 = 2b2 y además a − b es múltiplo de n.

10. Sea ABCDEF un hexágono convexo que no posee dos lados paralelos, tal que ∠AF B =
∠F DE, ∠DF E = ∠BDC y ∠BF C = ∠ADF . Pruebe que las rectas AB, F C y DE
son concurrentes si y sólo si las rectas AF , BE y CD son concurrentes.

22
Dı́a 3

11. Se tiene un triángulo ABC. Sea P un punto variable en el interior del triángulo ABC
tal que las rectas AP y CP cortan a los lados BC y AB en los puntos D y E, respecti-
vamente, de tal forma que el área del triángulo AP C sea igual al área del cuadrilátero
BDP E. Probar que la circunferencia circunscrita al triángulo BDE pasa por un punto
fijo distinto de B.

12. Cada punto del plano que tiene ambas coordenadas enteras es pintado de rojo, verde
o azul. Halle el menor entero positivo n que tiene la siguiente propiedad: sin importar
cómo se pintan los puntos, existe un triángulo de área n que tiene sus tres vértices del
mismo color.

13. Sea r un entero positivo y sea Nr el menor entero positivo tal que los números
 
Nr 2n
, n = 0, 1, 2, 3, . . . ,
n+r n
 
r 2r
son todos enteros. Probar que Nr = .
2 r

23
Dı́a 4

14. Sea Z+ el conjunto de los enteros positivos, halle todas las funciones f : Z+ → Z+
tales que
m2 + f (n) | mf (m) + n,
para todos los enteros positivos m y n.

15. Sea n un entero positivo y considere la secuencia a1 , a2 , . . . , an de enteros positivos.


Extienda periódicamente dicha secuencia hasta obtener una sucesión infinita a1 , a2 , . . .
definiendo an+i = ai para todo i ≥ 1. Si

a1 ≤ a2 ≤ · · · ≤ an ≤ a1 + n

y
aai ≤ n + i − 1, para i = 1, 2, . . . , n

a) Pruebe que ai ≤ n + i − 1 para i = 1, 2, . . . , n.


b) Pruebe que a1 + · · · + an ≤ n2 .

16. Sea n un entero positivo y A un subconjunto de {1, 2, 3, . . . , n}. Una A-partición de


n en k partes es una representación de n como la suma n = a1 + · · · + ak , donde
las partes a1 , . . . , ak pertenecen a A y no necesariamente son distintas. El número de
partes diferentes en una tal partición es el número de elementos distintos en el conjunto
{a1 , a2 , . . . , ak }.
Decimos que una A-partición de n en k partes es óptima si no existe una A-partición
de n en r partes
√ con r < k. Pruebe que cualquier A-partición óptima de n contiene
3
como máximo 6n partes diferentes.

24
Selectivo IMO 2013

Dı́a 1

1. Algunos enteros positivos están escritos en una fila. Una operación consiste en lo si-
guiente: Alicia escoge dos números adyacentes a y b, tales que a > b y a está a la
izquierda de b, y reemplaza la pareja (a, b) por una de las siguientes parejas: (b + 1, a)
ó (a − 1, a). Pruebe que Alicia sólo puede realizar un número finito de operaciones.

2. Sea a ≥ 3 un número real, y P un polinomio de grado n y coeficientes reales. Pruebe


que al menos uno de los siguientes números es mayor o igual que 1:

|a0 − P (0)|, |a1 − P (1)|, |a2 − P (2)|, . . . , |an+1 − P (n + 1)|.

3. Un punto P está en el lado AB de un cuadrilátero convexo ABCD. Sea ω la circunfe-


rencia inscrita del triángulo CP D e I el centro de ω. Se sabe que ω es tangente a las
circunferencias inscritas de los triángulos AP D y BP C en K y L, respectivamente.
Sea E el punto de intersección de las rectas AC y BD, y F el punto de intersección de
las rectas AK y BL. Pruebe que los puntos E, I, F son colineales.

25
Dı́a 2

4. Sea A un punto fuera de una circunferencia ω. Por A se trazan dos rectas que cortan a
ω, la primera corta a ω en B y C, mientras que la segunda corta a ω en D y E (D está
entre A y E). La recta que pasa por D y es paralela a BC intersecta a ω en F 6= D,
y la recta AF intersecta a ω en T 6= F . Sean M el punto de intersección de BC y
ET , N el simétrico de A con respecto a M , y K el punto medio de BC. Pruebe que el
cuadrilátero DEKN es cı́clico.

5. Determine todos los enteros m ≥ 2 que tienen la siguiente propiedad: Cualquier entero
m m n

n tal que 3
≤n≤ 2
es un divisor del número m−2n
.

6. A y B juegan con N ≥ 2012 monedas y 2012 cajas distribuidas alrededor de una circun-
ferencia. Al inicio A distribuye las monedas en las cajas de tal forma que haya al menos
1 moneda en cada caja. Luego ellos realizan sus jugadas en el orden B, A, B, A, . . . de
acuerdo a las siguientes reglas:

B en su jugada pasa una moneda de cada caja a una caja adyacente.


A en su jugada escoge algunas monedas que no estuvieron involucradas en la
jugada anterior (realizada por B) y que están en cajas diferentes. Luego A pasa
cada moneda escogida a una caja adyacente.

El objetivo de A es asegurar que haya al menos 1 moneda en cada caja después de


cualquier jugada suya, sin importar cómo juega B y cuántas jugadas hubo en total.
Halle el menor N para el cual A puede lograr su objetivo.

26
Selectivo IMO 2012

Dı́a 1

1. a) Encuentre una función f : R → R tal que:


x2 − x
f (f (x)) = · f (x) + 2 − x, ∀x ∈ R. (∗)
2

b) De todas las funciones f : R → R que cumplen la condición (∗), halle los posibles
valores de f (2).

2. Sean a, b, c las longitudes de los lados de un triángulo, y ha , hb , hc las longitudes de


1
las alturas de dicho triángulo relativas a los lados a, b, c, respectivamente. Si t ≥ 2
es
un número real, demuestre que existe un triángulo de lados:

t · a + ha , t · b + hb , t · c + hc .

3. Suponga que 1000 estudiantes están sentados alrededor de una circunferencia. Pruebe
que existe un entero k, con 100 ≤ k ≤ 300, para el cual existe un grupo de 2k estu-
diantes consecutivos tal que la primera mitad del grupo contiene el mismo número de
mujeres que la segunda mitad.

27
Dı́a 2

4. Se tiene una retı́cula triangular infinita en la cual la distancia entre dos puntos adya-
centes siempre es igual a 1:

Se escogen los puntos A, B y C de la retı́cula que son los vértices de un triángulo


equilátero de lado L, en cuyos lados ya no hay más puntos de la retı́cula. Demuestra
L2 − 1
que en el interior del triángulo ABC hay exactamente puntos de la retı́cula.
2

5. Sea ABCD un parelogramo tal que ∠ABC > 90◦ y L la recta perpendicular a BC que
pasa por B. Suponga que el segmento CD no corta a L. De todas las circunferencias
que pasan por C y D, hay una que es tangente a L en P y hay otra que es tangen-
te a L en Q (P 6= Q). Si M es el punto medio de AB, demuestre que ∠P M D = ∠QM D.

6. Sea p un número primo impar. Para cada entero a, definimos el número:


a a2 ap−1
Sa = + + ··· + .
1 2 p−1
Sean m y n números enteros tales que:
m
S3 + S4 − 3S2 = ,
n
demuestre que m es múltiplo de p.

28
Selectivo IMO 2011

Dı́a 1

1. Sea Z+ el conjunto de los enteros positivos. Encuentre todas las funciones f : Z+ → Z+


que satisfacen la siguiente condición: Para cada entero positivo n, existe algún entero
positivo k tal que
X k
fi (n) = kn,
i=1

donde f1 (n) = f (n) y fi+1 (n) = f (fi (n)), para i ≥ 1.

2. Sea A1 A2 · · · An un polı́gono convexo. El punto P es escogido en el interior del polı́gono


de tal manera que sus proyecciones P1 , P2 , . . . , Pn sobre las rectas A1 A2 , A2 A3 , . . . , An A1 ,
respectivamente, pertenecen a los lados del polı́gono (y no a sus prolongaciones). Prue-
be que si X1 , . . . , Xn son puntos arbitrarios que pertenecen a los lados A1 A2 , . . . , An A1 ,
respectivamente, se cumple la desigualdad:
 
X1 X2 Xn X1
máx ,..., ≥ 1.
P1 P2 P n P1

3. Sean a, b números enteros, y sea P (x) = ax3 + bx. Dado un entero positivo n, decimos
que el par ordenado (a, b) es n-bueno si n | P (m) − P (k) implica que n | m − k para
todos los enteros m, k. Decimos también que el par (a, b) es muy bueno si (a, b) es
n-bueno para infinitos enteros positivos n.

a) Encuentre un par (a, b) que sea 51-bueno, pero que no sea muy bueno.
b) Demuestre que todos los pares 2010-buenos también son muy buenos.

29
Dı́a 2

4. Sea ABC un triángulo acutángulo en el que se han trazado las alturas AA1 , BB1 y
CC1 . Sea A2 un punto del segmento AA1 tal que ∠BA2 C = 90◦ ; se definen análoga-
mente los puntos B2 y C2 . Sea A3 el punto de intersección de los segmentos B2 C y
BC2 ; se definen análogamente los puntos B3 y C3 . Pruebe que los segmentos A2 A3 ,
B2 B3 y C2 C3 son concurrentes.

5. En cierto planeta, existen 2N paı́ses (N ≥ 4). Cada paı́s tiene una bandera de N
metros de largo y 1 metro de alto, compuesta por N cuadrados de 1 metro de lado,
cada cuadrado está pintado de rojo o azul. No hay dos paı́ses que tengan la misma
bandera.
Decimos que un conjunto de N banderas es diverso si estas banderas pueden ser or-
denadas en un tablero de N × N de tal manera que todos los N cuadrados que están
sobre la diagonal principal tengan el mismo color. Determine el menor entero positivo
M tal que entre cualesquiera M banderas distintas, existen N banderas que forman
un conjunto diverso.

6. Sean a1 , a2 , . . . , an números reales, con n ≥ 3, tales que a1 + a2 + · · · + an = 0 y además


2ak ≤ ak−1 + ak+1 para k = 2, 3, . . . , n − 1.

Determine el menor número λ(n), tal que para todo k ∈ {1, 2, . . . , n} se cumpla que

|ak | ≤ λ(n) · máx{|a1 |, |an |}.

30
Selectivo IMO 2010

Dı́a 1

1. En cualquier triángulo no equilátero P QR, se cumple que su ortocentro, baricentro


y circuncentro pertenecen a una misma recta, esa recta recibe el nombre de recta de
Euler del triángulo P QR.
Sea ABC un triángulo acutángulo y F un punto en el interior del triángulo tal que

∠AF B = ∠BF C = ∠CF A = 120◦ .

Pruebe que las rectas de Euler de los triángulos AF B, BF C y CF A son concurrentes.

2. Un entero positivo N es llamado balanceado, si N = 1 o si N puede ser representado


como el producto de una cantidad par de números primos, no necesariamente diferentes.
Dados los enteros positivos a y b, considere el polinomio P definido por P (x) = (x +
a)(x + b).

a) Pruebe que existen dos enteros positivos distintos a y b tales que los números
P (1), P (2), . . . , P (50) son balanceados.
b) Pruebe que si P (n) es balanceado para todo entero positivo n, entonces a = b.

3. Cinco jarras vacı́as e idénticas de 2 litros de capacidad están ubicadas sobre los vérti-
ces de un pentágono regular. Cenicienta y su madrastra malvada realizan el siguiente
procedimiento por rondas: Al inicio de cada ronda, la madrastra toma un litro de agua
de un rı́o cercano y distribuye su contenido arbitrariamente en las 5 jarras. Luego Ce-
nicienta escoge un par de jarras vecinas, vierte todo su contenido en el rı́o, y las regresa
a su lugar. Luego la siguiente ronda empieza. El objetivo de la madrastra es conseguir
que una de las jarras rebalse. El objetivo de Cenicienta es prevenir que eso suceda. ¿La
madrastra puede asegurar que una de las jarras rebalse?

31
Dı́a 2

4. Sean x, y, z reales positivos tales que x + y + z = 1. Pruebe que


1 + xy 1 + yz 1 + zx
+ + ≥ 5.
x+y y+z z+x

5. Sea N el conjunto de los enteros positivos. Para cada subconjunto X de N definimos


el conjunto ∆(X ) como el conjunto de todos los números |m − n|, donde m y n son
elementos de X , es decir:

∆(X ) = {|m − n| ; m, n ∈ X }.

Sean A y B dos conjuntos infinitos y disjuntos cuya unión es N.

a) Pruebe que el conjunto ∆(A) ∩ ∆(B) tiene infinitos elementos.


b) Pruebe que existe un subconjunto infinito C de N tal que ∆(C) es un subconjunto
de ∆(A) ∩ ∆(B).

6. Los lados AD y BC de un cuadrilátero ABCD (tal que AB no es paralelo a CD) se


intersectan en un punto P . Los puntos O1 y O2 son los circuncentros, y los puntos H1
y H2 son los ortocentros de los triángulos ABP y DCP , respectivamente. Sean E1 y
E2 los puntos medios de los segmentos O1 H1 y O2 H2 , respectivamente. Pruebe que la
recta perpendicular a CD que pasa por E1 , la recta perpendicular a AB que pasa por
E2 y la recta H1 H2 son concurrentes.

32
Dı́a 3

7. Sean a, b, c reales positivos tales que a + b + c = 1. Pruebe que


1 1 1 11
+ + + 3(ab + bc + ca) ≥
a+b b+c c+a 2

8. Dado un cuadrilátero cı́clico ABCD, las diagonales AC y BD se intersectan en E y


las rectas AD y BC se intersectan en F . Los puntos medios de AB y CD son G y H,
respectivamente. Pruebe que EF es tangente en E a la circunferencia que pasa por los
puntos E, G y H.

9. Halle todos los enteros positivos n para los cuales existe una secuencia de enteros
positivos a1 , a2 , . . . , an tal que:

a2k + 1
ak+1 = −1
ak−1 + 1
para todo k con 2 ≤ k ≤ n − 1.

33
Selectivo IMO 2009

Dı́a 1

1. Pruebe que existen infinitas ternas (x, y, z) de números reales que satisfacen las ecua-
ciones:
x2 + y = y 2 + z = z 2 + x
tales que x, y, z son distintos dos a dos.

2. En el congreso se forman 3 comisiones disjuntas de 100 congresistas cada una. Cada


pareja de congresistas se conocen o no se conocen entre sı́. Demuestra que existen dos
congresistas, de comisiones distintas, tales que la tercera comisión contiene 17 congre-
sistas que conocen a ambos, o 17 congresistas que no conocen a ninguno de ellos.

3. Sea ABCDEF un hexágono convexo que no tiene ningún par de lados paralelos. Se
sabe que, para todo punto P interior al hexágono, el valor de:

Área[ABP ] + Área[CDP ] + Área[EF P ]

es constante. Pruebe que los triángulos ACE y BDF tienen el mismo baricentro.

34
Dı́a 2

4. Demuestre que existen 2009 números naturales consecutivos, tales que cada uno de
ellos cumpla que el cociente de su mayor divisor primo entre su menor divisor primo
es un número racional mayor que 20.

5. Sea C la circunferencia inscrita al triángulo ABC, la cual es tangente a los lados BC,
AC, AB en los puntos A0 , B 0 , C 0 , respectivamente. En C se toman los puntos distintos
K y L tales que:

∠AKB 0 + ∠BKA0 = ∠ALB 0 + ∠BLA0 = 180◦ .

Pruebe que los puntos A0 , B 0 , C 0 están a igual distancia de la recta KL.

6. Sea N el conjunto de los enteros positivos. Determine si existe o no un par de funciones


(f, g), ambas definidas de N a N, que satisfagan las siguientes condiciones a la vez:

(i) Ambas funciones son estrictamente crecientes, es decir, para todo x, y ∈ N con
x < y, se cumple que f (x) < f (y) y g(x) < g(y).
(ii) Para todo x ∈ N se cumple que f (g(g(x))) < g(f (x)).

35
Selectivo IMO 2008

Dı́a 1

1. En el triángulo ABC, sea I el centro de la circunferencia inscrita y sea IA el centro


de la circunferencia ex-inscrita opuesta al vértice A. Sea LA la recta que pasa por los
ortocentros de los triángulos BIC y BIA C. Se definen de forma análoga las rectas LB
y LC . Pruebe que las rectas LA , LB , LC pasan por un mismo punto.

2. Encuentre todas las funciones f : R → R que satisfacen la ecuación:


f (2f (x) + y) = f (f (x) − f (y)) + 2y + x,

para cualesquiera números reales x, y.

3. Dado cualquier número natural n, consideremos la secuencia (ai ), 1 ≤ i ≤ 2n, definida


de la siguiente forma:

a2k−1 = −k, 1≤k≤n


a2k = n − k + 1, 1 ≤ k ≤ n.

Decimos que el par (b, c) es bueno si se cumplen las dos condiciones siguientes:

i) 1 ≤ b < c ≤ 2n,
Xc
ii) aj = 0
j=b

Si B(n) es el número de pares buenos correspondientes a n, demuestre que existen


infinitos n para los cuales B(n) = n.

36
Dı́a 2

4. Sean S1 y S2 circunferencias no concéntricas tales que S1 está en el interior de S2 . Sea


K un punto variable de S1 . La recta tangente a S1 en el punto K corta a S2 en los
puntos A y B. Sea M el punto medio del arco AB que está en el semiplano determinado
por AB que no contiene a S1 . Encontrar el lugar geométrico del punto simétrico de M
respecto a K.

5. Cuando cortamos una cuerda en dos partes, decimos que el corte es especial si las dos
partes obtenidas tienen longitudes distintas. Cortamos una cuerda, de 2008 de longitud,
en dos partes de longitudes enteras y escribimos estos dos enteros en la pizarra. Luego
cortamos una de las partes en dos partes de longitudes enteras y escribimos estos dos
enteros en la pizarra. El proceso termina cuando todas las partes tienen 1 de longitud.

a) Halle el mı́nimo número posible de cortes especiales.


b) Pruebe que, para todos los procesos que tienen el mı́nimo número posible de cor-
tes especiales, la cantidad de enteros distintos en la pizarra es siempre la misma.

6. Para cada entero positivo n, sea d(n) el número de divisores positivos de n. Decimos
a2 b
que un entero positivo es feliz si se puede expresar en la forma , donde a > b > 0
a−b
son enteros. Además, decimos que un entero positivo m es malvado si no existe un
entero feliz n tal que d(n) = m. Demuestre que si un entero positivo es feliz y malvado,
a la vez, entonces es una potencia de 4.

37
Selectivo IMO 2007

Dı́a 1

1. Sea P un punto interior de un semicı́rculo de diámetro AB (el ángulo ∠AP B es ob-


tuso). La circunferencia inscrita al triángulo ABP es tangente a los lados AP y BP
en los puntos M y N , respectivamente. La recta M N corta a la semicircunferencia en
los puntos X y Y . Pruebe que la medida del arco XY es igual a la medida del ángulo
∠AP B.

1 1 1
2. Sean a, b, c tres números reales positivos tales que a + b + c ≥ + + . Pruebe que
a b c
3 2
a+b+c≥ + .
a + b + c abc

3. Sea N un número natural que se puede expresar en la forma a2 + b2 + c2 , donde a, b y c


son enteros divisibles por 3. Pruebe que N se puede expresar en la forma x2 + y 2 + z 2 ,
donde x, y y z son enteros y ninguno de ellos es divisible por 3.

4. En un tablero de 2007 × 2007 casillas, se colorearon de negro n casillas de modo que


se cumple la siguiente condición: No hay tres casillas negras formando alguna de las
siguientes figuras:

Pero si se pinta de negro cualquier otra casilla, ya no se cumple la condición dada.


Determine el menor valor de n para el cual esto es posible.

38
Dı́a 2

5. Sean k un entero positivo y P (x) un polinomio de coeficientes enteros. Pruebe que


existe un entero positivo n tal que P (1) + P (2) + · · · + P (n) es divisible por k.

6. En un triángulo acutángulo ABC, con CA 6= CB, los puntos A1 y B1 son los puntos
de tangencia de las circunferencias exinscritas relativas a los lados CB y CA, respec-
tivamente, e I es el incentro. La recta CO interseca a la circunferencia circunscrita
al triángulo ABC en el punto P . La recta que pasa por P y es perpendicular a CP
interseca a la recta AB en el punto Q. Pruebe que las rectas QO y A1 B1 son paralelas.

7. Sea T un conjunto de 2007 puntos en el plano donde no hay tres puntos colineales. Sea
P cualquier punto de T . Pruebe que el número de triángulos que contienen a P en su
interior y cuyos vértices están en T es par.

8. Sean a, b y c los lados de un triángulo. Probar que


√ √ √
b+c−a c+a−b a+b−c
√ √ √ +√ √ √ +√ √ √ ≤3
b+ c− a c+ a− b a+ b− c

39
Exámenes Selectivos para la
Olimpiada Iberoamericana de
Matemática

Comisión de Olimpiadas de la Sociedad Matemática Peruana

Edición: Jorge Tipe


Versión: enero 2020
Prólogo

En la Olimpiada Iberoamericana de Matemática participan alrededor de 22 paı́ses de Su-


damérica, Centroamérica, el Caribe y la Penı́nsula Ibérica. Cada paı́s está representado por
un equipo de 4 estudiantes. Pueden participar alumnos que no hayan cumplido 18 años al
año anterior de la realización de la Olimpiada, además, un alumno puede participar como
máximo dos veces en una Olimpiada Iberoamericana.
La sede cambia anualmente, es ası́ que las últimas ediciones de la Olimpiada Iberoamericana
se han realizado en:

2019: Guanajuato, México.

2018: La Rábida, España y Monte Gordo, Portugal.

2017: Iguazú, Argentina.

2016: Antofagasta, Chile.

2015: Mayagüez, Puerto Rico.

2014: San Pedro Sula, Honduras.

2013: Ciudad de Panamá, Panamá.

2012: La Paz, Bolivia.

2011: San José, Costa Rica.

2010: Asunción, Paraguay.

En el Perú, la Comisión de Olimpiadas de la Sociedad Matemática Peruana está a cargo de


la selección de los alumnos, y con este fin se toman exámenes selectivos algunos meses antes
de la realización de la olimpiada. En los últimos años el proceso selectivo consta de 2 o 3
exámenes selectivos.
Si encuentran un error, tienen una sugerencia para aclarar la redacción de un problema, o
si tienen cualquier otra consulta con respecto a este archivo, me pueden enviar un correo a
jorgetipe@gmail.com por lo cual estaré muy agradecido. Iré actualizando este archivo con el
paso del tiempo. Por ejemplo, si consigo exámenes de años anteriores a los aquı́ presentados.

Jorge Tipe

Comisión de Olimpiadas
de la Sociedad Matemática Peruana

1
Selectivo Ibero 2019

Dı́a 1

1. Dado un número entero positivo n, encuentre el número de sucesiones de longitud n


cuyos elementos son únicamente 0 y 1 tales que el número de veces que aparece 0 0 en
la sucesión es igual al número de veces que aparece 1 1 en la sucesión. A manera de
ejemplo, cuando n = 4 tenemos exactamente 4 sucesiones con esa caracterı́stica: 0 0 1 1,
0 1 0 1, 1 0 1 0 y 1 1 0 0.

2. Sean a1 , a2, , a3 , . . . y b1 , b2, , b3 , . . . dos sucesiones infinitas de números enteros positivos


en progresión aritmética, y sea m un entero positivo. Suponga que existen ı́ndices
i, j ≥ 1 tales que (ai , bj ) = 1 e ı́ndices k, l ≥ 1 tales que (ak , b l ) = m. Pruebe que para
todo divisor entero positivo d de m, existen ı́ndices s, t ≥ 1 tales que (as , bt ) = d.
Nota: Si a y b son números enteros positivos, (a, b) denota al máximo común divisor
de a y b.

3. Sea M un número real positivo tal que para toda sucesión infinita a1 , a2, , a3 , . . . de
números reales positivos y para todo número real m < M , existe un ı́ndice n ≥ 1 tal
que
a1 + a2 + a3 + · · · + an + an+1 > m an .
¿Cuál es el mayor valor posible de M ?

2
Dı́a 2

4. Ana y Bety juegan en un tablero de 2018 × 2019 casillas blancas. En cada turno una de
ellas escoge un conjunto conexo de 9 casillas blancas y pinta esas 9 casillas de negro.
Ana empieza y luego alternan los turnos. Pierde quien ya no puede realizar su turno.
Determine quién de las dos tiene una estrategia ganadora.
Nota: Un conjunto de casillas es llamado conexo si para cualesquiera dos casillas P y
Q del conjunto, existe una sucesión C1 , C2 , . . . , Ck de casillas del conjunto tales que:

C1 = P y Ck = Q.
Las casillas Ci y Ci+1 tienen un lado común para todo i ∈ {1, 2, . . . , k − 1}.

5. Sea Γ el circuncı́rculo de un triángulo acutángulo ABC y sea G su baricentro. Sean


M y N los puntos medios de los lados AC y AB respectivamente. Sea D el pie de
la perpendicular trazada desde A al lado BC. Una circunferencia ω que pasa por los
puntos M y N es tangente a Γ en un punto X (X 6= A). Pruebe que los puntos X, D
y G son colineales.

6. Encuentre todas las triplas (a, b, c) de números enteros con c ≥ 0 tales que:
2ab no es un cuadrado perfecto.
bn + c
Para todo entero positivo n el número es entero.
an + 2 n

3
Selectivo Ibero 2018

Pre-selectivo

1. Sean p y q números reales. Sabiendo que existen números reales positivos a, b y c,


distintos dos a dos, tales que

a2 b2 c2
p= + + ,
(b − c)2 (c − a)2 (a − b)2
1 1 1
q= + + ,
(b − c)2 (c − a)2 (a − b)2

calcule el valor de
a b c
2
+ 2
+
(b − c) (c − a) (a − b)2
en términos de p y q.

2. Sea ABC un triángulo con AB = AC y sea D el pie de la altura trazada desde


A a BC. Sea P un punto en el interior del triángulo ADC tal que ∠AP B > 90◦ y
∠P AD + ∠P BD = ∠P CD. Las rectas CP y AD se cortan en Q y las rectas BP y AD
se cortan en R. Sea T un punto en el segmento AB tal que ∠T RB = ∠DQC y sea S un
punto en la prolongación del segmento AP (del lado de P ) tal que ∠P SR = 2∠P AR.
Pruebe que RS = RT .

3. Para cada entero positivo m, sea P (m) el producto de todos los dı́gitos de m. Se define
la sucesión a1 , a2 , a3 , . . . de la siguiente manera:

a1 es un entero positivo menor que 2018,


an+1 = an + P (an ) para cada entero n ≥ 1.

Pruebe que para todo entero n ≥ 1 se cumple que an ≤ 22018 .

4. Encuentre todos los enteros n ≥ 2 para los cuales es posible dividir cualquier triángulo
T en triángulos T1 , T2 , . . . , Tn y elegir medianas m1 , m2 , . . . , mn , una en cada uno de
dichos triángulos, de manera que estas n medianas tengan igual longitud.

4
Dı́a 1

5. Encuentre todos los enteros positivos a, b y c tales que los números


a+1 b+1 c+1
, y
b c a
sean enteros positivos.

6. Encuentre todos los números reales a para los que existe una función f : R → R tal
que
f (x + f (y)) = f (x) + abyc,
para cualesquiera números reales x y y.
Nota: Si x es un número real, bxc denota al mayor entero que es menor o igual que x.

7. Se tiene un conjunto finito de puntos en el plano, donde cada punto está pintado de
alguno de n colores distintos (n ≥ 4). Se sabe que hay al menos un punto de cada color
y que la distancia entre cualquier par de puntos de colores distintos es menor o igual
a 1. Pruebe que es posible escoger 3 colores de forma que, al remover del plano todos
los puntos de esos colores, el conjunto de puntos restantes puede ser cubierto con un
1
cı́rculo de radio √ .
3

5
Dı́a 2

8. Una nueva pieza de ajedrez llamada mamut se mueve como un alfil (es decir, de forma
diagonal), pero solamente en 3 de las 4 direcciones posibles. Distintos mamuts en el
tablero pueden tener direcciones faltantes distintas. Encuentre el número máximo de
mamuts que es posible colocar en un tablero de ajedrez 8 × 8 de forma que ningún
mamut pueda ser atacado por otro.

9. Sea Γ el circuncı́rculo de un triángulo ABC con AB < BC, y sea M el punto medio
del lado AC. La mediatriz del lado AC corta a Γ en los puntos X e Y (X en el arco
˙ El circuncı́rculo del triángulo BM Y corta a la recta AB en P (P 6= B) y a la
ABC).
recta BC en Q (Q 6= B). Los circuncı́rculos de los triángulos P BC y QBA se cortan
en R (R 6= B). Pruebe que los puntos X, B y R son colineales.

10. Determine si existe una sucesión infinita a1 , a2 , a3 , . . . de enteros positivos que satisface
las siguientes condiciones:

(i) Cada entero positivo aparece exactamente una vez en la sucesión.


(ii) Para todo entero positivo n, el número de divisores positivos de

nann+1 + (n + 1)an+1
n

es divisible por n.

6
Selectivo Ibero 2017

Dı́a 1

1. Sean C1 y C2 circunferencias tangentes internas en un punto A, con C2 en el interior


de C1 . Sea BC una cuerda de C1 que es tangente a C2 . Pruebe que la razón entre la
longitud de BC y el perı́metro del triángulo ABC es constante, es decir, no depende
de la elección de la cuerda BC que se elija para construir el triángulo.

2. Determine si existe un entero positivo n tal que n2 + 11 es un número primo y n + 4


es un cubo perfecto.

3. Se tiene una mesa en forma de polı́gono regular de 1000 lados, donde cada lado tiene
longitud 1. En uno de los vértices hay un escarabajo (considere que este vértice es fi-
jo). Todos los 1000 vértices deben ser enumerados, en algún orden, usando los números
1, 2, . . . , 1000 de tal forma que el escarabajo esté en el vértice 1. El escarabajo solo
se puede mover a lo largo del borde de la mesa y siempre lo hace en sentido horario.
El escarabajo se mueve del vértice 1 al vértice 2, y ahı́ se detiene. Luego se mueve
del vértice 2 al vértice 3, y ahı́ se detiene. Ası́ sucesivamente, hasta que el escarabajo
termina su recorrido en el vértice 1000. Halle el número de formas en que se pueden
asignar los números a los vértices para que la longitud total del recorrido del escarabajo
sea 2017.

7
Dı́a 2

4. Tenemos un conjunto de 2n enteros positivos cuya suma es múltiplo de n. Una ope-


ración consiste en elegir n de ellos y sumarles el mismo entero positivo a todos ellos.
Demuestre que, partiendo de los 2n números iniciales, podemos conseguir que todos
sean iguales, realizando como máximo 2n − 1 operaciones.

5. Sea ABCD un trapecio de bases AD y BC, con AD > BC, cuyas diagonales se cortan
en el punto E. Sean P y Q los pies de las perpendiculares trazadas desde E hacia los
lados AD y BC, respectivamente, con P y Q en los segmentos AD y BC, respectiva-
mente. Sea I el incentro del triángulo AED y sea K el punto de intersección de las
rectas AI y CD. Si AP + AE = BQ + BE, demuestre que AI = IK.

6. Para cada entero positivo k sea S(k) la suma de los dı́gitos de k en el sistema decimal.
Pruebe que existe un entero positivo k, que no tiene ningún dı́gito 9 en su representación
decimal, tal que  2017 
S 224 k = S(k).

8
Selectivo Ibero 2016

Dı́a 1

1. Sea ABC un triángulo rectángulo isósceles, recto en C. Los puntos M y N pertenecen


a los segmentos AC y BC, respectivamente, tales que M N = BC. Sea ω una circun-
ferencia que es tangente al segmento AB y que pasa por los puntos M y N . Halle el
lugar geométrico del centro de ω, conforme M y N varı́an.

2. Considere un conjunto de 100 personas para el cual existe un entero no negativo m


(fijo) que tiene la siguiente propiedad: En cualquier subconjunto de 7 personas, hay
exactamente m parejas de personas que se conocen. Determine todos los valores de m
para los cuales esta situación es posible.
Aclaración: A conoce a B si y sólo si B conoce a A. Las parejas que se mencionan en
el enunciado no necesariamente son disjuntas.

3. Decimos que un punto del plano cartesiano es entero si sus dos coordenadas son núme-
ros enteros. Dados dos puntos enteros P y Q, decimos que P y Q son amigos si el
segmento P Q no contiene otros puntos enteros aparte de P y Q. Determine si existe
o no una secuencia de 100 puntos enteros P1 , P2 , . . . , P100 que cumpla las siguientes
condiciones a la vez:

a) Pi y Pi+1 son amigos para 1 ≤ i ≤ 99; además, P100 y P1 son amigos.


b) Las únicas parejas de amigos son los mencionados en la parte (a).
c) No hay tres puntos de la secuencia que sean colineales.

9
Dı́a 2

4. Halle todas las sucesiones a0 , a1 , a2 , . . . que cumplen las siguientes condiciones a la vez:
an+1 = 2a2n − 1, para todo entero n ≥ 0.
a0 es un número racional.
ai = aj para algunos enteros no negativos i, j, con i 6= j.

5. Sean n y k enteros positivos tales que n ≥ 2k. Se tiene una fila de n casillas y en cada
casilla se va a escribir una letra A o una letra B, de tal forma que haya exactamente
k letras A que tienen a su derecha una letra B. Determine, en función de n y k, de
cuántas formas se puede hacer esto.

6. En un cuadrilátero convexo ABCD se tiene que ∠ABC = ∠BCD = 120◦ , M es el


punto medio del segmento BC, y O es el punto de intersección de las diagonales AC
y BD. Sea K el punto de intersección de M O y AD. Si ∠BKC = 60◦ , pruebe que
∠BKA = ∠CKD = 60◦ .

10
Dı́a 3

7. Sea ABC un triángulo con ∠A > 90◦ , en el que se ha trazado la altura CH y las
medianas AM y BN . Si la circunferencia de diámetro AM es tangente a la recta CH,
pruebe que la circunferencia de diámetro BN también es tangente a la recta CH.

8. Sean a, b, c, d números reales que pertenecen al intervalo [1, 2]. Pruebe que
abcd
|(a − b)(b − c)(c − d)(d − a)| ≤ ,
4
y determine en qué casos ocurre la igualdad.

9. En el plano cartesiano han sido dibujados 2016 cuadrados de lado 2. Los lados de los
cuadrados son paralelos a los ejes cartesianos. Además, se sabe que el centro de cual-
quier cuadrado no es un punto interior de algún otro cuadrado. Sea R un rectángulo
que contiene a todos esos cuadrados y cuyos lados son paralelos a los ejes cartesianos.
Determine el menor valor posible del perı́metro de R.

11
Selectivo Ibero 2015

Dı́a 1

1. En el triángulo acutángulo ABC, tal que AB 6= AC, sea D el pie de la perpendicu-


lar trazada desde A a la recta BC. Sean E y F los puntos medios de los segmentos
AD y BC, respectivamente. Si G es el pie de la perpendicular trazada desde B a la
recta AF , pruebe que EF es tangente a la circunferencia circunscrita al triángulo GF C.

2. Determine el mayor número real λ que tiene la siguiente propiedad: Para todo intervalo
no vacı́o [a, b] que no contiene ningún entero, existe un entero positivo N tal que
el intervalo [N a, N b] no contiene ningún entero y además la longitud del intervalo
[N a, N b] es mayor o igual que λ.
Aclaración: El intervalo [a, b] es el conjunto de todos los números reales x tales que
a ≤ x ≤ b. La longitud del intervalo [a, b] es b − a.

3. Sea n un entero positivo. Demuestre que el conjunto {1, 2, 3, . . . , 2n } se puede particio-


nar en dos conjuntos tales que ninguno de ellos contenga 2n elementos en progresión
aritmética.

12
Dı́a 2

4. Sea A un conjunto de seis enteros positivos distintos dos a dos cuya suma es 90. Es-
cribimos esos números en las caras de un cubo (uno en cada cara). En una jugada
escogemos tres caras del cubo que tienen un vértice en común, y sumamos 1 a cada
uno de los números de esas caras. Determine para cuántos conjuntos A es posible es-
cribir los elementos de A en las caras del cubo y luego realizar algunas jugadas para
conseguir que todos los números sean iguales.

5. Sea N el conjunto de los enteros positivos. Definimos la función f : N → N como


2k
f (2k − 1) = 2k y f (2k) = k + , donde mk es el mayor divisor impar del entero
mk
positivo k. Denotemos las funciones f 0 (x) = x y f n (x) = f (f n−1 (x)) para todo entero
positivo n. Si x y y son enteros positivos tales que

f x−1 (2x ) · f y (1) = 2014,

determine todos los posibles valores de x + y.

6. Sea ABCD un cuadrilátero inscrito en una circunferencia de centro O. En los lados


AB y CD se consideran los puntos F y E, respectivamente, tales que EO = F O.
Las rectas AD y BC cortan a la recta EF en los puntos M y N , respectivamente.
Finalmente, el punto P es el simétrico de M con respecto al punto medio del segmento
AE. Demostrar que los triángulos F BN y CEP son semejantes.

13
Dı́a 3

7. Hallar el mı́nimo valor de abc sobre todas las ternas (a, b, c) de enteros positivos que
satisfacen la ecuación a−2 + b−2 = c−2 .

8. Sea ABC un triángulo tal que AB > AC, con circuncı́rculo ω. Se trazan las tangentes
a ω por B y C, y éstas se intersectan en P . La perpendicular a AP por A corta a BC
en R. Sea S un punto sobre el segmento P R tal que P S = P C.

a) Demuestre que las rectas CS y AR se cortan sobre ω.


b) Sea M el punto medio de BC y Q el punto de intersección de CS y AR. Si ω y
el circuncı́rculo de AM P se cortan en un punto J (J 6= A), demuestre que P , J
y Q están alineados.

9. Si A = {a0 , a1 , a2 , . . .} es una sucesión infinita de dı́gitos no nulos (es decir 1 ≤ ai ≤ 9


para cada i ≥ 0), se dice que A es alternante si para cada término distinto del primero,
este término es estrictamente mayor que sus dos vecinos o estrictamente menor que sus
dos vecinos. Se dice que A es alternante de orden k si la subsucesión {akm : m ≥ 0} es
alternante.
Determine el mayor entero positivo M para el cual existe una sucesión A = {a0 , a1 , a2 , . . .}
de dı́gitos no nulos tal que A es alternante de orden k para cada k = 1, 2, . . . , M .

14
Selectivo Ibero 2014

Dı́a 1

1. Las circunferencias C1 y C2 se intersectan en los puntos distintos A y B. Las rectas


tangentes a C1 que pasan por A y B se intersectan en T . Sea M un punto sobre C1 que
está fuera de C2 . La recta M T intersecta nuevamente a C1 en C, la recta M A intersecta
nuevamente a C2 en K y la recta AC intersecta nuevamente a la circunferencia C2 en
L. Pruebe que la recta M C pasa por el punto medio del segmento KL.

2. Sea n ≥ 4 un número entero. Se tiene dos tableros de n × n. Cada tablero contiene


los números del 1 al n2 inclusive, un número por casilla, ordenados de forma arbitraria
en cada tablero. Un movimiento consiste en intercambiar dos filas o dos columnas del
primer tablero (no se puede realizar movimientos en el segundo tablero). Pruebe que
es posible realizar una secuencia de movimientos de tal modo que para todo 1 ≤ i ≤ n
y 1 ≤ j ≤ n, el número que está en la i-ésima fila y j-ésima columna del primer table-
ro sea diferente al número que está en la i-ésima fila y j-ésima columna del segundo
tablero.

3. Un entero positivo n es llamado especial si existen enteros a > 1 y b > 1 tales que
n = ab + b. ¿Existe un conjunto de 2014 enteros positivos consecutivos que contenga
exactamente 2012 números especiales?

15
Dı́a 2

4. Determine el mı́nimo valor de


x2014 + 2x2013 + 3x2012 + 4x2011 + · · · + 2014x + 2015,

donde x es un número real.

5. La circunferencia inscrita del triángulo acutángulo ABC es tangente a los lados AB y


AC en K y L, respectivamente. La altura AH intersecta a las bisectrices de los ángulos
∠ABC y ∠BCA en los puntos P y Q, respectivamente. Sean ω1 y ω2 las circunferencias
circunscritas de los triángulos KP B y LQC, respectivamente. Sea M el punto medio
de AH. Si M está fuera de ω1 y ω2 , pruebe que las longitudes de las tangentes desde
M a ω1 y ω2 son iguales.

6. Determine el mayor entero positivo k para el cual existe un grafo simple G de 2014
vértices que cumple las siguientes condiciones a la vez:

a) G no contiene triángulos.
b) Para cada i entre 1 y k, inclusive, al menos un vértice de G tiene grado i.
c) Ningún vértice de G tiene grado mayor que k.

Aclaración: En un grafo simple dos vértices cualesquiera están unidos a lo más por
una arista y no hay lazos, es decir, no hay un vértice que esté unido con sı́ mismo. Un
triángulo es un ciclo formado por tres aristas.

16
Selectivo Ibero 2013

Dı́a 1

1. Sea (a, b, c) una terna formada por enteros positivos distintos tales que a+b+c = 2013.
Un paso consiste en reemplazar la terna (x, y, z) por la terna (y+z−x, z+x−y, x+y−z).
Pruebe que, empezando con la terna (a, b, c), después de 10 pasos obtendremos una
terna que contiene al menos un número negativo.

2. Sea n un entero positivo y X un conjunto con |X| = n. Se dice que una familia F de
subconjuntos de X (distintos entre sı́) es especial si existen A, B ∈ F tales que A ⊂ B
y |B − A| = 1.
Determine el menor m tal que cualquier familia F con |F| > m es especial.
Aclaración: Si C es un conjunto, |C| denota la cantidad de elementos que contiene C;
si S es una familia, |S| denota la cantidad de subconjuntos que contiene S. Para los
conjuntos M y N , M − N denota el conjunto formado por todos los elementos que
están en M pero no están N .

3. Sean a, b, c reales positivos tales que el siguiente sistema de ecuaciones:


a2 x + b 2 y + c 2 z = 1
xy + yz + zx = 1

tiene solución única (x, y, z) en el conjunto de los números reales. Pruebe que a, b, c
representan los lados de un triángulo.

17
Dı́a 2

4. Cada casilla de un tablero 8 × 8 contiene exactamente un número 1 ó −1. Considere


todas las posiciones de la figura:

sobre el tablero (la figura cubre exactamente cuatro casillas del tablero, la figura se
puede rotar pero todas sus casillas deben permanecer dentro del tablero). Una posición
de la figura es llamada pobre si la suma de los cuatro números que cubre es diferente
de cero. Determine el menor número de posiciones pobres que puede haber en total.

5. Sea C una circunferencia fija, A y B son puntos fijos de C (con A 6= B) y ` una recta
−→ −−→
que no corta a C. Sea P un punto variable de C tal que los rayos AP y BP cortan a `
en D y E, respectivamente. Pruebe que la circunferencia de diámetro DE es siempre
tangente a dos circunferencias fijas conforme P varı́a en C.

6. a) Determine si existe un subconjunto infinito S de los enteros positivos tal que


S 6= Z+ y tal que para cada entero positivo n ∈
/ S, exactamente n elementos de
S son coprimos con n.
b) Determine si existe un subconjunto infinito S de los enteros positivos tal que para
cada n ∈ S, exactamente n elementos de S son coprimos con n.
Aclaración: Z+ denota el conjunto de los enteros positivos.

18
Selectivo Ibero 2012

Dı́a 1

1. Para una sucesión finita S, sea m(S) la mayor cantidad de veces que se repite un
término de S. Para una sucesión A de 2n + 1 números reales distintos a1 , a2 , . . . , a2n+1
considere la sucesión A0 cuyos elementos son:
a1 + a2 a1 + a2 + a3 a1 + a2 + · · · + a2n+1
a1 , , ,..., .
2 3 2n + 1

Hallar el mayor valor que puede tomar m(A0 ), sobre todas las sucesiones a1 , a2 , . . . , a2n+1
de números reales distintos.

2. Los cuadrados de una cuadrı́cula infinita son pintados de blanco y negro, alternando
los colores como en un tablero de ajedrez. Todos los cuadrados son de lado 1.
Un polı́gono P (no necesariamente convexo) de área A y perı́metro B está formado
por algunos cuadrados (es decir, los lados del polı́gono están incluidos en las lı́neas de
4A + B
la cuadrı́cula). Probar que P no contiene más de cuadrados negros.
8

3. Sea ABCD un cuadrilátero convexo tal que AB · CD = AD · BC. Probar que


∠BAC + ∠CBD + ∠DCA + ∠ADB = 180◦ .

19
Dı́a 2

4. Sean a1 , a2 , a3 , . . . , a8 enteros positivos distintos tales que ninguno de ellos es un cubo


perfecto. Considere todos los productos de la forma ai aj , con 1 ≤ i < j ≤ 8, y sea N
la cantidad de esos productos que son cubos perfectos. Halle el mayor valor posible de
N.

5. Un conjunto A de sucesiones de ceros y unos, todas de longitud 5, se llama universal si


se cumple que toda sucesión infinita de ceros y unos contiene 5 términos consecutivos
que forman un elemento de A. Determine la menor cantidad de elementos que puede
tener un conjunto universal.

6. Pruebe que existen infinitos enteros positivos compuestos n tales que (3n−1 − 2n−1 ) es
múltiplo de n.

20
Selectivo Ibero 2011

Dı́a 1

1. Encuentre todos los números enteros m > 1 para los cuales la suma de los cubos de
los dı́gitos de cualquier múltiplo de m también es múltiplo de m.

2. Sea ABC un triángulo acutángulo escaleno y sea H su ortocentro. Las rectas BH y


CH cortan a AC y AB en D y E, respectivamente. La circunferencia circunscrita al
triángulo ADE corta a la circunferencia circunscrita al triángulo ABC en un punto F
distinto de A. Pruebe que las bisectrices interiores de los ángulos ∠BF C y ∠BHC se
cortan en un punto sobre el segmento BC.

3. Sean p y q enteros positivos. Halle el menor entero positivo m (en función de p y q)


para el cual entre cualesquiera m enteros distintos del intervalo [−p, q] existen tres cuya
suma es cero.

21
Dı́a 2

4. Sean a1 , a2 , a3 , a4 , a5 cinco números reales cuya suma es 0 y tales que |ai − aj | ≤ 1,


para todos los ı́ndices i, j ∈ {1, 2, 3, 4, 5}. Pruebe que
6
a21 + a22 + a23 + a24 + a25 ≤
5

5. En un tablero de n × n casillas, cada casilla tiene escrito un signo ((+)) o un signo ((−)).
Una operación consiste en escoger una fila o una columna y remplazar cada signo en
dicha fila o columna por su signo opuesto. Se sabe que al inicio hay exactamente 2
signos ((−)) en el tablero (y todos los otros son ((+))) y que después de algunas ope-
raciones hay exactamente 9 signos ((−)) en el tablero. Determine el menor y el mayor
valor posible de n para que ocurra esto.

6. Sea n un número entero mayor o igual que 3 y sea L una colección de n rectas del plano
en posición general (es decir, en L no hay dos rectas paralelas ni tres concurrentes). Para
cada terna de rectas de L consideramos el disco circular abierto inscrito en el triángulo
determinado por esas rectas. Halle, en función de n, el número de tales discos que no
son intersectados por ninguna recta de L.
Aclaración. Un disco circular abierto no incluye el borde del disco.

22
Selectivo Ibero 2010

Dı́a 1

1. Sea n un entero positivo. Sabemos que el conjunto In = {1, 2, . . . , n} tiene exactamen-


te 2n subconjuntos, en consecuencia, hay 8n ternas ordenadas (A, B, C), donde A, B
y C son suconjuntos de In . Para cada una de estas ternas consideramos el número
|A ∩ B ∩ C|. Pruebe que la suma de los 8n números considerados es múltiplo de n.

Aclaración: |Y | denota la cantidad de elementos del conjunto Y .

2. Para cada entero positivo k sea S(k) la suma de los dı́gitos de k en el sistema decimal.
Encuentre todos los enteros positivos N para los cuales existen enteros positivos a, b, c,
coprimos dos a dos, tales que:

S(ab) = S(bc) = S(ca) = N.

3. Sean C1 y C2 dos circunferencias concéntricas de centro O, de tal forma que el radio de


C1 es menor que el radio de C2 . Sea P un punto distinto de O que está en el interior
de C1 , y L una recta que pasa por P y corta a C1 en A y B. El rayo OB corta a C2
en C. Determine el lugar geométrico que determina el circuncentro del triángulo ABC
conforme L varı́a.

23
Dı́a 2

4. Halla el menor entero k > 1 para el cual nk − n es múltiplo de 2010 para todo entero
positivo n.

5. El trapecio ABCD de bases AB y CD está inscrito en una circunferencia Γ. Sea X un


punto variable del arco AB
d que no contiene a C ni a D. Sea Y el punto de intersección
XZ AY
de AB y DX, y sea Z el punto del segmento CX tal que = . Demuestre que
XC AB
la medida del ángulo ∠AZX no depende de la elección de X.

6. En un tablero de n × n, el conjunto de todas las casillas que están ubicadas en la


diagonal principal del tablero o debajo de ella, es llamado n-escalera. Por ejemplo, en
la siguiente figura se muestra una 3-escalera:

¿De cuántas formas se puede dividir una 99-escalera en algunos rectángulos, que tengan
sus lados sobre lı́neas de la cuadrı́cula, de tal forma que todos los rectángulos tengan
áreas distintas?

24
Selectivo Ibero 2009

Dı́a 1

1. Un conjunto P tiene la siguiente propiedad: “ Para cualquier entero positivo k, si p es


un factor primo de k 3 + 6, entonces p pertenece a P ”. Pruebe que P es infinito.

2. Un mago y su asistente hacen su presentación frente a un público de muchas personas.


En el escenario hay un tablero 8 × 8, el mago se venda los ojos, y luego el asistente va
invitando a personas del público para que escriban los números 1, 2, 3, 4, . . . , 64 en las
casillas que quieran (un número por casilla) hasta completar los 64 números. Después
el asistente tapa dos casillas adyacentes, a su elección. Finalmente, el mago se saca la
venda de los ojos y tiene que “adivinar”qué número hay en cada casilla que tapó el
asistente. Explicar cómo armaron este truco.
Aclaración: Dos casillas son adyacentes si tienen un lado en común.

3. Sean M, N, P los puntos medios de los lados AB, BC, CA de un triángulo ABC. Sea
X un punto fijo en el interior del triangulo M N P . Las rectas L1 , L2 , L3 que pasan por
el punto X son tales que L1 intersecta al segmento AB en el punto C1 y al segmento
AC en el punto B2 ; L2 intersecta al segmento BC en el punto A1 y al segmento BA
en el punto C2 ; L3 intersecta al segmento CA en el punto B1 y al segmento CB en el
punto A2 . Indique cómo construir las rectas L1 , L2 , L3 de manera que la suma de las
áreas de los triángulos A1 A2 X, B1 B2 X y C1 C2 X sea mı́nima.

25
Dı́a 2

4. Sea ABC un triángulo tal que AB < BC. Se traza la altura BH con H en AC. Sean
I el incentro del triángulo ABC y M el punto medio de AC. Si la recta M I interseca
a BH en el punto N , pruebe que BN < IM .

5. Sean a, b, c enteros positivos cuyo máximo común divisor es 1. Determine si siempre


existe un entero positivo n tal que, para todo entero positivo k, el número 2n no es un
divisor de ak + bk + ck .

6. Sea P un conjunto de n ≥ 2 puntos distintos en el plano, que no contiene ninguna


terna de puntos alineados. Sea S el conjunto de todos los segmentos que tienen por
extremos a puntos de P . Dados dos segmentos s1 , s2 ∈ S, escribiremos s1 ⊗ s2 si la
intersección de s1 con s2 es un punto distinto de los extremos de s1 y s2 .
Demostrar que  un segmento s0 ∈ S tal que el conjunto {s ∈ S | s0 ⊗ s} tiene
 existe
1 n−2
al menos elementos.
15 2

26
Selectivo Ibero 2008

1. Para cada número entero m > 1, sea p(m) el menor divisor primo de m. Si a y b son
enteros mayores que 1, tales que

a2 + b = p(a) + [p(b)]2 ,

demostrar que a = b.

2. Sea ABCD un cuadrilátero convexo y sean P y Q puntos en el interior de ABCD tal


que P QDA y QP BC son cuadriláteros cı́clicos. Suponga que exista un punto E en el
segmento P Q tal que ∠P AE = ∠QDE y ∠P BE = ∠QCE. Pruebe que el cuadrilátero
ABCD es cı́clico.

3. En el plano coordenado, considere el conjunto S de todos los puntos cuyas coordena-


das son números enteros. Para un entero positivo k, dos puntos distintos A, B ∈ S son
llamados k-amigos si existe un punto C ∈ S tal que el área del triángulo ABC es igual
a k. Un conjunto T ⊂ S es llamado k-completo si dos puntos cualesquiera de T son
k-amigos. Halle el menor entero positivo k para el cual existe un conjunto k-completo
que tiene más de 200 elementos.

27
Selectivo Ibero 2007

1. Resolver en el conjunto de los números reales, el sistema:


x(3y 2 + 1) = y(y 2 + 3)
y(3z 2 + 1) = z(z 2 + 3)
z(3x2 + 1) = x(x2 + 3).

2. Encontrar todos los enteros positivos m y n que satisfacen la igualdad


n5 + n4 = 7m − 1.

3. Se tiene un triángulo acutángulo ABC. Consideremos el cuadrado A1 A2 A3 A4 que tiene


un vértice en AB, un vértice en AC y dos vértices (A1 y A2 ) en BC y sea xA = ∠A1 AA2 .
Análogamente se definen xB y xC . Probar que xA + xB + xC = 90◦ .

4. Cada una de las casillas de un tablero de 15 × 15 tiene un cero. En cada paso se escoge
una fila o una columna, borramos todos los números de ella y luego escribimos los
números del 1 al 15 en las celdas vacı́as, en un orden arbitrario. Encontrar la suma
máxima posible de los números en el tablero que se puede lograr después de un número
finito de pasos.

28

También podría gustarte